Download as pdf or txt
Download as pdf or txt
You are on page 1of 130

All India Mock Test 02 Report Print this page

Question 1. + 2.0 - 0.66

The opening and last sentences of the Preamble, “We, the people……………adopt, enact and give to
ourselves this Constitution”, signifies:

A. the sovereign nature of the State

  B. the political power is ultimately vested with the people

C. the idea of welfare State

D. the government is limited by the Constitution

Question Analytics
5160 USERS 3608 USERS 1552 USERS 32.8 SECS

ATTEMPTED  SOLVED CORRECTLY  SOLVED INCORRECTLY  YOUR TIME

61.7 SECS 0.0 SECS

 AVG. SOLVING TIME  FASTEST SOLVING TIME

Explanation :

The Preamble of the Constitution of India serves as a brief introductory statement that sets
out the guiding purpose, principles and philosophy of the Constitution.
The idea of the following things are given by the Preamble which includes:
Source of the Constitution
Nature of Indian State
A statement of its objectives
Date of its adoption
The opening and last sentences of the preamble: “We, the people... adopt, enact and give to
ourselves this Constitution” signifies the power is ultimately vested in the hands of the
people.

Therefore, option (b) is the correct answer.


Relevance: On the 75th Republic Day, the Government of India shared on social media a copy of the
original Preamble to the Constitution of India.
Question 2. + 2.0 - 0.66

Consider the following:


1. Procedure established by law
2. Emergency provisions
3. Appointment of Governors

4. Parliamentary form of government


How many of the above features of the Constitution of India have been taken from the Government
of India Act, 1935?

 A. Only one

B. Only two

 C. Only three

D. All four

Question Analytics
4733 USERS 1025 USERS 3708 USERS 36.5 SECS

ATTEMPTED  SOLVED CORRECTLY  SOLVED INCORRECTLY  YOUR TIME

56.62 SECS 0.0 SECS

 AVG. SOLVING TIME  FASTEST SOLVING TIME

Explanation :

The Constitution of India draws upon a variety of sources, including historical documents,
legal precedents, constitutions of various other countries, as well as the Government of India
Act, 1935.
The concept of ‘procedure established by law’ enumerated in Article 21 (Protection of Life
and Personal Liberty) of the Constitution of India is influenced by the Japanese Constitution.
So, point 1 is not correct.
The provisions related to the features such as the federal scheme, office of the Governor,
judiciary, public service commissions, emergency provisions and administrative details have
been derived from the Government of India Act, 1935.
The emergency provisions have been borrowed from Constitutions of Weimar
(Germany) and Government of India Act, 1935. So, point 2 is correct.
The Governor of a State in India is appointed by the President of India on the advice of the
Union Cabinet. This feature in the Constitution was borrowed from the Constitution of
Canada. So, point 3 is not correct.
The provisions related to the features namely Parliamentary Government, rule of law,
legislative procedure, single citizenship, cabinet system, prerogative writs, Parliamentary
privileges and bicameralism have been borrowed from the British Constitution. So, point 4 is
not correct.

So, only one of the above features of the Constitution of India have been taken from the
Government of India Act, 1935.

Therefore, option (a) is the correct answer.

Knowledge Box

Government of India Act, 1935 and Emergency Provisions:

The Indian Constitution followed the Government of India Act, 1935 which embodied
emergency provisions in Section 45 for the Centre and in Section 93 for the Provinces.
It empowered the Chief Executive to declare an emergency. The Act of 1935 was meant to
provide provincial autonomy in India. But it also had emergency provisions which put
restrictions on the autonomy of provinces. It involved questions of relations between centre
and provincial units.
The Act empowered the Centre to control and intervene in provincial matters if emergency
arises due to war or internal disturbance and make provision for carrying out administration
if the machinery of ministerial government failed to function.
In times of emergency due to war or internal disturbances, the Governor General was armed
with power to make laws on all matters, even those in the provincial list. The Governor
General could declare an emergency in provinces.
The Governor General of India had in fact specific responsibility concerning peace and
tranquillity of India or any part thereof. Even he could act independently in the
provincial government through the Governor who was also empowered to function
in the respective provinces and was the main source of information of the Governor-
General.

Question 3. + 2.0 - 0.66

Which one of the following statements is correct?


A. India's dominion status ended with the Indian Independence Act of 1947.

 B. India's dominion status continued until the adoption of the Constitution of India in 1950.

C. India remained a dominion until the first general elections in 1951.

D. India's dominion status ended with the withdrawal of British troops in 1949.

Question Analytics
4869 USERS 3136 USERS 1733 USERS 210.1 SECS

ATTEMPTED  SOLVED CORRECTLY  SOLVED INCORRECTLY  YOUR TIME

41.05 SECS 0.0 SECS

 AVG. SOLVING TIME  FASTEST SOLVING TIME

Explanation :

Dominions were autonomous communities within the British Empire which were “equal in
status” but had an “allegiance to the Crown”.
It meant that King George VI continued to reign as the Emperor of India and Lord
Mountbatten was the first Governor-General of the country after independence.
Jawahar Lal Nehru was sworn in as the Prime Minister and unelected Indian nationalist
leaders were administered oaths in the name of the British King-Emperor.
Though Indian Independence Day is celebrated on August 15, 1947, in reality, that was the
moment when India and Pakistan acquired dominion status. While India remained a
dominion till 1950, Pakistan retained the status till 1956.
It meant that a British field marshall led the Indian army and judges appointed by the British
continued to be part of the High Courts and the Federal Court.
It was only on January 26, 1950, that India broke free from the monarchy and transformed
into a sovereign democratic republic after its Constitution came into effect.

Therefore, option (b) is the correct answer.

Question 4. + 2.0 - 0.66

Which of the following provisions of the Constitution of India directly embody social justice as its
principle?

1. Article 21A

2. Article 23
3. Article 38

4. Article 48A
5. Article 50

Select the correct answer using the code given below:

A. 1 and 2 only

 B. 1, 2 and 3 only

C. 3, 4 and 5 only

D. 1, 2, 3, 4 and 5

Question Analytics
4225 USERS 2630 USERS 1595 USERS 36.8 SECS

ATTEMPTED  SOLVED CORRECTLY  SOLVED INCORRECTLY  YOUR TIME

66.84 SECS 0.0 SECS

 AVG. SOLVING TIME  FASTEST SOLVING TIME

Explanation :

Social justice holds that all people should have equal access to wealth, health, well-being,
privileges and opportunity.
The Constitution of India has ingrained the spirit of social justice into its provisions. The
Preamble mentions 3 kinds of justice- social, political and economic. All of these are
interconnected. Social justice and equality are complementary to each other. The 'welfare state'
concept & socialistic features of the Constitution and Indian economy are also a step towards
social justice.
Many Fundamental Rights and Directive Principles of State Policy embody the principle of
social justice. Some of them are:
Right to Equality - Article 14, 16, 17 and 18
Right to Education - Article 21A. So, point 1 is correct.
Cultural and Educational Rights - Article 29 and 30
Right against Exploitation - Article 23 and 24. So, point 2 is correct.
Promotion of social welfare - Article 38. So, point 3 is correct.
Distribution of resources as per common good - Article 39
Just and humane working conditions - Article 42
Promotion of educational and economic interests of the Scheduled castes, Scheduled
tribes and other weaker sections - Article 46
Article 48A states that ‘’The State shall endeavour to protect and improve the environment
and to safeguard the forests and wildlife of the country.’’ So, point 4 is not correct.
Article 50 of the Constitution of India states that ‘’the State shall take steps to separate the
judiciary from the executive in the public services of the State.’’ Neither of these provisions
directly promote the idea of social justice. So, point 5 is not correct.

Therefore, option (b) is the correct answer.


Relevance: Recently, the World Day of Social Justice was celebrated.

Question 5. + 2.0 - 0.66

Which one of the following is not a feature of ‘asymmetric federalism’ in India?

A. Existence of All India Services

 B. Flexibility in Constitutional Amendment

C. Integrated nature of Judiciary

 D. Seventh Schedule of the Constitution of India

Question Analytics
4703 USERS 1818 USERS 2885 USERS 102.2 SECS

ATTEMPTED  SOLVED CORRECTLY  SOLVED INCORRECTLY  YOUR TIME

67.48 SECS 0.0 SECS

 AVG. SOLVING TIME  FASTEST SOLVING TIME

Explanation :

Asymmetric federalism refers to federalism based on unequal powers and relationships in


political, administrative and fiscal spheres between the units constituting a federation.
Asymmetry in the arrangements in a federation can be viewed in both vertical (between
Centre and States) and horizontal (among the States) senses.
Even when the Constitution guarantees near equal powers to the States, in the overall federal
system, the Centre dominates in political, administrative as well as fiscal spheres.
Examples of the asymmetric nature of federalism in India are:
All-India Services: In India, the Centre and the States have their separate public
services. However, there are All-India Services which are common to both the Centre
and the States. The members of these services are recruited and trained by the Centre
which also possesses ultimate control over them. Thus, these services violate the
principle of federalism under the Constitution.
Flexibility in the Constitutional Amendment: The process of Constitutional
amendment is less rigid in India in comparison to what is found in other federations.
The bulk of the Constitution can be amended by the unilateral action of the
Parliament, either by a simple majority or by a special majority. Further, the power to
initiate an amendment to the Constitution lies only with the Centre. In the United States
of America (USA), the States can also propose an amendment to the Constitution.
Integrated Judiciary: The Constitution has established an integrated judicial system
with the Supreme Court at the top and the State High Courts below it. This single
system of courts enforces both the Central laws as well as the State laws. In the USA,
on the other hand, there is a double system of courts whereby the federal laws are
enforced by the federal judiciary and the state laws by the state judiciary.
Division of Powers: The Constitution of India has divided the powers between the Centre and
the States in terms of the Union List, State List and Concurrent List in the Seventh
Schedule. The Union List consists of 98 subjects (originally 97), the State List 59 subjects
(originally 66) and the Concurrent List 52 subjects (originally 47). Both the Centre and the
states can make laws on the subjects of the concurrent list, but in case of a conflict, the Central
law prevails. The residuary subjects (ie, which are not mentioned in any of the three lists) are
given to the Centre. It is a federal feature.

Therefore, option (d) is the answer.


Relevance: Recently, the Kerala Finance Minister criticised the Union Government for imposing
financial disparities on Kerala, emphasising the importance of cooperative federalism.

Question 6. + 2.0 - 0.66

Which of the following limits the authority and jurisdiction of the Parliament?

1. Preamble
2. Seventh Schedule

3. Judicial Review
4. Fundamental Rights

Select the correct answer using the code given below:

A. 1 and 3 only

B. 2 and 4 only

 C. 1, 3 and 4 only

 D. 2, 3 and 4 only

Question Analytics
5017 USERS 3644 USERS 1373 USERS 32.3 SECS

ATTEMPTED  SOLVED CORRECTLY  SOLVED INCORRECTLY  YOUR TIME

50.97 SECS 0.0 SECS

 AVG. SOLVING TIME  FASTEST SOLVING TIME

Explanation :

The Parliament in India is not a sovereign body, it functions within the bounds of a written
Constitution. In India, unlike the United Kingdom, we have a written constitution which puts
necessary limitations on all organs of the state. Parliament can amend some parts of the
Constitution but only to the degree bestowed upon it by the Constitution.

Preamble: It is neither a source of power to legislature nor a prohibition upon the powers of
legislature. It is non-justiciable, that is, its provisions are not enforceable in courts of law. So,
point 1 is not correct.
Seventh Schedule: Under Article 246 of the Constitution, this Schedule deals with the
division of powers between the Union and the States. It divides the legislative subjects into
Union, Concurrent and State lists. The Parliament is explicitly prohibited from legislating on
matters in the State List. So, point 2 is correct.
Judicial Review: The Indian Parliament, being the architect of the Constitution, doesn’t enjoy
unchecked authority. For a law to be considered valid, it must undergo judicial review. If a law
is found to be inconsistent with the constitutional provisions, the court has the authority to
declare it void. This means that the judiciary assesses the constitutionality of legislative
enactments, limiting parliamentary sovereignty in India. So, point 3 is correct.
Fundamental Rights: It guarantees Fundamental Rights, which are justiciable, meaning they
can be enforced through legal proceedings. Parliament is expressly restricted from enacting
laws that violate these Fundamental Rights. So, point 4 is correct.

Therefore, option (d) is the correct answer.


Relevance: The Vice-President recently questioned the Basic Structure Doctrine on the grounds that
it limits Parliamentary authority.

Question 7. + 2.0 - 0.66

Consider the following statements with reference to the making of the Constitution of India:
1. It is handwritten in both English and in Hindi.

2. Rani Lakshmibai of Jhansi is the only female figure illustrated prominently in the Constitution.
3. The Bull Seal found in the Indus Valley is the first pictorial representation in the Constitution.
How many of the above statements are correct?

A. Only one

 B. Only two

 C. All three

D. None

Question Analytics
3963 USERS 588 USERS 3375 USERS 26.9 SECS

ATTEMPTED  SOLVED CORRECTLY  SOLVED INCORRECTLY  YOUR TIME

45.65 SECS 0.0 SECS

 AVG. SOLVING TIME  FASTEST SOLVING TIME

Explanation :

The Constitution of India is not just the lengthiest written Constitution in the world but also the
most richly illustrated, with all 22 parts carrying hand-painted images and its pages adorned
with elaborate borders.
Two copies of the Constitution, one in English and one in Hindi, are handwritten and
bear the paintings. While the Constitution was handwritten by calligrapher Prem Behari
Narain Raizada, the paintings were conceived and implemented in Santiniketan by
artist-pedagogue Nandalal Bose and his team. So, statement 1 is correct.
The Preamble page has intricate patterns sketched by Beohar Rammanohar Sinha and
bears his signature, while Dinanath Bhargava sketched the National Emblem, the Lion
Capital of Ashoka.
The only female figure illustrated prominently in the Constitution, Rani Lakshmibai of
Jhansi, is sketched in her armour as she shares the page with Tipu Sultan, the king of Mysore,
in Part XVI of the Constitution. Chhatrapati Shivaji and Guru Gobind Singh are featured in
Part XV. So, statement 2 is correct.
The Bull Seal, excavated from the Indus Valley region, is the first pictorial representation in
the Constitution, appearing in ‘Part I: The Union and its Territory’. So, statement 3 is correct.

So, all three of the above statements are correct.

Therefore, option (c) is the correct answer.


Relevance: India celebrated Samvidhan Divas, also known as Constitution Day, on November 26 to
commemorate the adoption of the Constitution in 1949.

Question 8. + 2.0 - 0.66

Consider the following statements:

Statement-I: The provisions of the Tenth Schedule, with respect to voting against the instruction of
the party, do not apply to Rajya Sabha elections.

Statement-II: The Constitution of India provides that the voting in elections to the Rajya Sabha shall
be through an open ballot.

Which one of the following is correct in respect of the above statements?

Both Statement–I and Statement–II are correct and Statement–II is the correct
A.
explanation for Statement–I

Both Statement–I and Statement–II are correct and Statement–II is not the correct
B.
explanation for Statement–I

 C. Statement–I is correct but Statement–II is incorrect

D. Statement–I is incorrect but Statement–II is correct

Question Analytics
4502 USERS 1792 USERS 2710 USERS 51.3 SECS

ATTEMPTED  SOLVED CORRECTLY  SOLVED INCORRECTLY  YOUR TIME

64.38 SECS 0.0 SECS


 AVG. SOLVING TIME  FASTEST SOLVING TIME

Explanation :

The 52nd Constitutional Amendment Act of 1985 introduced the ‘anti-defection’ law and
added the Tenth Schedule to the Constitution of India. This Schedule provides that a member
of a House of Parliament or State Legislature who voluntarily gives up the membership of their
political party or votes against the instructions of their party in a House are liable for
disqualification from such House. This instruction with respect to voting is issued by the
‘whip’ of a party.
However, the elections to Rajya Sabha are not treated as a proceeding within the
Legislative Assembly. The Election Commission of India, drawing reference to Supreme
Court judgments, had specified that the provisions of the Tenth Schedule, with respect
to voting against the instruction of the party, will not be applicable for a Rajya Sabha
election. Furthermore, political parties cannot issue any ‘whip’ to its members for such
elections. So, statement 1 is correct.
In order to control the Members of Legislative Assemblies (MLAs) from cross-voting, an
amendment to the Representation of the People Act, 1951 (and not the Constitution of
India) was carried out in 2003. It provided that the voting in elections to Rajya Sabha shall
be through an open ballot. The MLAs of political parties are required to show their ballot
paper to the authorised agent of their Party. Not showing the ballot paper to the authorised
agent or showing it to anyone else will disqualify the vote. Independent MLAs are barred from
showing their ballots to anyone. So, statement 2 is not correct.

So, statement–I is correct but Statement–II is incorrect.


Therefore, option (c) is the correct answer.

Relevance: Recently, the Speaker of the Maharashtra Legislative Assembly refused to disqualify 16
Members of the Legislative Assembly (MLAs).

Question 9. + 2.0 - 0.66

A ‘Parliamentary democracy’ is characterised by:

A. strict separation of the executive and legislative organs

 B. appointment of the executive branch by the legislature

C. direct elections of the representative to the executive office


 D. fusion of executive and legislative powers into a single body

Question Analytics
4949 USERS 1275 USERS 3674 USERS 28.3 SECS

ATTEMPTED  SOLVED CORRECTLY  SOLVED INCORRECTLY  YOUR TIME

56.71 SECS 0.0 SECS

 AVG. SOLVING TIME  FASTEST SOLVING TIME

Explanation :

Parliamentary democracy is characterised by the fusion of executive and legislative powers


into a single body. The executive, the Cabinet Ministers, sit as members of Parliament and
play a double role in the fusion of executive and legislative powers.
Those who recommend legislation as members of the Cabinet also vote on the same legislation
as members of the governing legislature. Typically, the government is the Cabinet and the rest
of the members of the majority party or coalition of parties in the Parliament.
Parliamentary democracy means collegial executive. Although the Prime Minister is the chief
executive, he or she is not a singular executive. The collegial executive is a collection of
Ministers (Cabinet) who must make decisions as a group and must be in general agreement
before legislation is recommended or policies are proposed. The Ministers are both
individually and collectively responsible to the Parliament for their actions. Parliamentary
democracy also means a democracy based on party responsibility.

Therefore, option (d) is the correct answer.

Question 10. + 2.0 - 0.66

Consider the following bills:

1. Money Bill
2. Private Member Bill

3. Constitutional Amendment Bill

The President of India can use her/his Pocket Veto with respect to how many of the above?

 A. Only one

B. Only two
 C. All three

D. None

Question Analytics
4968 USERS 1978 USERS 2990 USERS 39.4 SECS

ATTEMPTED  SOLVED CORRECTLY  SOLVED INCORRECTLY  YOUR TIME

40.58 SECS 0.0 SECS

 AVG. SOLVING TIME  FASTEST SOLVING TIME

Explanation :

A Bill passed by the Parliament can become an Act only if it receives the assent of the
President of India. When a Bill is presented to the President for her/his assent, s/he has three
alternatives (under Article 111 of the Constitution of India):
S/He may give assent to the Bill
S/He may withhold assent to the Bill (Absolute Veto)
S/He may return the Bill (if it is not a Money Bill) for reconsideration of the Parliament
(Suspensive Veto). However, if the Bill is passed again by the Parliament with or
without amendments and again presented to the President, it is obligatory for the
President to give assent to the Bill.
With respect to Pocket Veto, the President neither ratifies nor rejects nor returns the Bill, but
simply keeps it pending for an indefinite period. This power of the President not to take any
action (either positive or negative) on the Bill is known as the Pocket Veto. The President can
exercise this veto power as the Constitution does not prescribe any time-limit within which he
has to take the decision with respect to a Bill presented to her/him for assent.
With respect to a Money Bill, the President may either give or withhold assent. By convention,
s/he gives the assent to the Bill as it is introduced in the Parliament based on her/his
recommendation. A Money Bill cannot be returned to the House by the President for
reconsideration. So, point 1 is not correct.
The President can withhold or keep it pending or return a Private Member Bill for the
reconsideration of the Parliament. So, point 2 is correct.
The President is bound to give her/his assent to the Constitutional Amendment Bill
passed by Parliament by the prescribed special majority and, where necessary, ratified by the
requisite number of State Legislatures. So, point 3 is not correct.
So, the President of India can use her/his Pocket Veto with respect to only one of the above
bills.

Therefore, option (a) is the correct answer.

Question 11. + 2.0 - 0.66

If the Gross National Product (GNP) of India is more than its Gross Domestic Product (GDP), it
necessarily implies an increase in:

  A. net factor income from abroad

B. depreciation of fixed assets

C. gross fixed capital consumption

D. output produced within India by non-residents

Question Analytics
4815 USERS 4215 USERS 600 USERS 98.4 SECS

ATTEMPTED  SOLVED CORRECTLY  SOLVED INCORRECTLY  YOUR TIME

52.28 SECS 0.0 SECS

 AVG. SOLVING TIME  FASTEST SOLVING TIME

Explanation :

Gross National Product (GNP) is the total value of all finished goods and services
produced by a country’s citizens in a given financial year, irrespective of their location. It also
measures the output generated by a country’s businesses located domestically or abroad.
It does not include foreign residents’ income earned within the country.
The Gross Domestic Product (GDP) is the final value of goods and services produced
in a given period of time within a country.
The basic distinction between GDP and GNP is the difference in estimating the production
output by foreigners in a country and by nationals outside of a country. It confines its analysis
of the economy to the geographical borders of the country, GNP extends it to also take
account of the net overseas economic activities performed by its residents. In other words,
GNP equals to GDP plus Net Factor Income From Abroad (NFIA).
NFIA is the difference between factor income earned by the residents who are temporarily
residing abroad and factor income earned by non-residents who are temporarily residing in our
country.

Therefore, option (a) is the correct answer.

Relevance: Recently, the Fitch Ratings raised India's economic growth forecast for next fiscal to 7
per cent.

Question 12. + 2.0 - 0.66

Consider the following statements:

The money multiplier in the economy will increase if there is:


1. an increase in the reserve requirement by the Central Bank

2. an increase in the currency deposit ratio


3. an increase in the deposit rates by banks

How many of the above statements are correct?

 A. Only one

 B. Only two

C. All three

D. None

Question Analytics
4497 USERS 1706 USERS 2791 USERS 36.4 SECS

ATTEMPTED  SOLVED CORRECTLY  SOLVED INCORRECTLY  YOUR TIME

64.85 SECS 0.0 SECS

 AVG. SOLVING TIME  FASTEST SOLVING TIME

Explanation :

Money multiplier shows the mechanism by which reserve money creates money supply in the
economy. It is dependent on two variables, namely currency deposit ratio and reserve deposit
ratio.
Reserve deposit Ratio, also known as Cash Reserve Ratio, is the percentage of deposits which
commercial banks are required to keep as cash according to the directions of the Central Bank.
If the reserve requirement as stipulated by the Central Bank increases, the Reserve Money
value will increase and the multiplier will fall. Similarly, if banks keep more money as excess
reserves, it will have an adverse effect on the money multiplier. So, statement 1 is not correct.
The currency deposit ratio shows the amount of currency that people hold as a proportion of
aggregate deposits. An increase in currency deposit ratio leads to a decrease in money
multiplier. So, statement 2 is not correct.
So, the banks can increase deposit rates to induce depositors to deposit more money
thereby leading to a decrease in cash to deposit ratio. This will in turn lead to a rise in
money multiplier. So, statement 3 is correct.
Increase in the banking habits of the people of the country will lead to more deposits in the
banks. This leads to an increase in money multiplier.

So, only one of the above statements is correct.

Therefore, option (a) is the correct answer.


Relevance: Liquidity conditions have remained tight in the banking system for some time due to
lower government spending, higher tax outflows and slower bank deposit growth.

Question 13. + 2.0 - 0.66

In an economy, the ‘Effective Capex’ means:

 A. capital expenditure less the grant-in-aid for creation of capital assets

B. capital expenditure less the depreciation value of the capital assets

C. expenditure incurred in the current accounting year to create revenue in a future

 D. total capital outlay of the government directed towards building of capital assets

Question Analytics
4100 USERS 1198 USERS 2902 USERS 43.7 SECS

ATTEMPTED  SOLVED CORRECTLY  SOLVED INCORRECTLY  YOUR TIME

56.92 SECS 0.0 SECS

 AVG. SOLVING TIME  FASTEST SOLVING TIME

Explanation :

Capital expenditure is the money spent by the government on the development of


machinery, equipment, building, health facilities, education, etc. It also includes the
expenditure incurred on acquiring fixed assets like land and investment by the government that
gives profits or dividends in future.
Effective Capital Expenditure is total capital outlay of the Government directed towards
building of capital assets and related expenditure. It is the sum of Capital Expenditure and
Grants-in-Aid for creation of Capital Assets (GIA Capital).

Therefore, option (d) is the correct answer.

Relevance: The Union Finance Minister in the Interim Budget of 2023-24 has announced that the
Effective Capital Expenditure of the Centre would be budgeted at ₹ 13.7 lakh crore i.e. 4.5% of GDP.

Question 14. + 2.0 - 0.66

Consider the following:


1. Nutrient Based Subsidy

2. Minimum Support Prices (MSP) for essential crops


3. Subsidy for purchase of machinery under centrally-sponsored schemes

4. Free electricity to farmers

How many of the above are considered indirect farm subsidies granted by the Government of India?

A. Only one

 B. Only two

 C. Only three

D. All four

Question Analytics
4626 USERS 1843 USERS 2783 USERS 113.0 SECS

ATTEMPTED  SOLVED CORRECTLY  SOLVED INCORRECTLY  YOUR TIME

64.91 SECS 0.0 SECS

 AVG. SOLVING TIME  FASTEST SOLVING TIME

Explanation :

Subsidies are provided through various schemes to the agricultural sector by the Central and
State Governments in order to promote the adoption of certain inputs/ machinery etc. in crop
cultivation.
Direct subsidies are monetary transfers by the government that reach the ultimate
beneficiary through a formal predetermined route.
The government provides direct subsidy through fixation of Minimum Support Price
(MSP) for essential crops and purchase of machinery, drip and sprinkler irrigation
under various centrally-sponsored schemes. So, points 2 and 3 are not correct.
Indirect subsidies by the Government are the ones that reach the farmers along with the use
of inputs. Therefore, these are highly correlated with the amount of use of inputs by farmers.
Generally, those farmers who use more inputs would enjoy higher subsidies.
The indirect subsidy is extended through provision of inputs such as irrigation,
fertiliser (Nutrient Based Subsidy) and power at prices much below their cost of
production (free electricity to farmers). So, points 1 and 4 are correct.

So, only two of the above are considered indirect farm subsidies granted by the Government of
India.

Therefore, option (b) is the correct answer.

Relevance: The Union Minister of Finance recently said that the World Trade Organisation (WTO)
should look at the issue of farm subsidies keeping in mind its impact on the food security needs of
emerging economies in the backdrop of the Russia-Ukraine war.

Question 15. + 2.0 - 0.66

Consider the following statements with reference to the interim budget:


1. The Constitution of India does not make any explicit provision for the interim budget.

2. It was first presented in the year of the first general election in India.

3. It deals only with the expenditure side of the government.


How many of the above statements are correct?

 A. Only one

 B. Only two

C. All three

D. None

Question Analytics
4371 USERS 1829 USERS 2542 USERS 56.3 SECS

ATTEMPTED  SOLVED CORRECTLY  SOLVED INCORRECTLY  YOUR TIME

49.39 SECS 0.0 SECS


49.39 SECS 0.0 SECS

 AVG. SOLVING TIME  FASTEST SOLVING TIME

Explanation :

The interim budget is a temporary financial plan announced before a new government is
set to come in after general elections. It can be understood as a provisional arrangement to
meet the expenditure of the government for a short period till the new government comes in
and makes the new full budget.
An interim budget is similar to a full budget but only has projections for a few months,
as opposed to a full financial year.
There is no constitutional provision for an interim budget. However, it has become a
common practice for outgoing governments ahead of elections. A ruling government can
alternatively choose to obtain the necessary funds for expenses through the vote-on-account
provision under Article 116 of the Constitution instead of presenting an interim budget. So,
statement 1 is correct.
India's first interim budget in 1947 was presented by the first Finance Minister R.K.
Shanmukham Chetty. It addressed economic challenges caused by partition, including food
grain shortage and high imports. So, statement 2 is not correct.
An interim budget contains both the revenue and expenditure details for the period until
the new government takes over. On the other hand, vote-on-account includes only the
government's expenditure. So, statement 3 is not correct.

So, only one of the above statements is correct.

Therefore, option (a) is the correct answer.

Relevance: The Interim Budget 2024 was presented by the Union Finance Minister recently.

Question 16. + 2.0 - 0.66

Which of the following measures can lead to a decline in Current Account Deficit of India?

1. Reduction in fertiliser and food subsidies


2. Diversification of the export markets

3. Reduction in customs duties on imports

4. Reduction in transfer costs of remittances

Select the correct answer using the code given below:


A. 1 and 2 only

B. 3 and 4 only

 C. 1, 2 and 4 only

 D. 1, 2, 3 and 4

Question Analytics
4406 USERS 2578 USERS 1828 USERS 72.1 SECS

ATTEMPTED  SOLVED CORRECTLY  SOLVED INCORRECTLY  YOUR TIME

86.0 SECS 0.0 SECS

 AVG. SOLVING TIME  FASTEST SOLVING TIME

Explanation :

The Current Account Deficit (CAD) is a measurement of a country’s trade where the value of
imports exceeds the value of its exports. It represents a country’s foreign transactions and,
like the capital account, is a component of its Balance of Payments (BoP). Several measures
can potentially lead to a decline in the CAD of India:
Fiscal Discipline: Fiscal consolidation measures such as reducing fertiliser and food
subsidies, controlling public expenditure and improving revenue generation can
help in narrowing the CAD. So, point 1 is correct.
Export Promotion: Implementing policies and strategies to boost exports can increase
foreign exchange inflows and reduce the trade deficit, which is a component of the
CAD. It can be achieved through trade promotion initiatives, improvement in
competitiveness, diversification of export markets and support to industries with
export potential. So, point 2 is correct.
Import Substitution: Encouraging domestic production of goods that are currently
being imported can reduce reliance on imports and subsequently narrow the trade deficit.
Promoting domestic industries, providing incentives, and improving the business
environment can support import substitution efforts. On the other hand, promotion of
imports by means of reducing customs duty and easing qualitative barriers can lead to
widening of the trade deficit. So, point 3 is not correct.
Increase in Remittances: Encouraging overseas workers to send more remittances back
to their home country can boost the current account balance. Measures such as
facilitating remittance channels, reducing transfer costs and promoting financial
inclusion can incentivize higher remittance inflows. So, point 4 is correct.
Current Account Management: This can involve controlling non-essential imports,
managing import costs, implementing effective exchange rate policies, and monitoring
capital flows to maintain stability.

Therefore, option (c) is the correct answer.

Relevance: Economists were prompted to lower the Current Account Deficit (CAD) at below 1% of
GDP for FY24 amidst narrowing of India’s trade deficit and increase in services exports.

Question 17. + 2.0 - 0.66

If the tax multiplier is 1 in an economy, then 1 percent tax cut will lead to:

  A. 1 percent boost to GDP

B. 10 percent boost to GDP

C. 1 percent reduction in GDP

D. no change in GDP

Question Analytics
3075 USERS 1014 USERS 2061 USERS 145.7 SECS

ATTEMPTED  SOLVED CORRECTLY  SOLVED INCORRECTLY  YOUR TIME

52.22 SECS 0.0 SECS

 AVG. SOLVING TIME  FASTEST SOLVING TIME

Explanation :

The tax multiplier is defined as the amount by which a change in taxes affects the Gross
Domestic Product (GDP). This tool allows the government to reduce (raise) taxes by the exact
amount required for GDP to rise (fall). This allows the government to implement a specific tax
change rather than an estimate.
Both public investments and tax cuts have multiplier effects on the economy. The capex
multiplier in India is around 2.5 while the tax multiplier is closer to 1. So, a one-rupee tax
cut can provide only a one-rupee boost to GDP, while every rupee spent on capex will boost
GDP by 2.5x.
The Union Budget of 2024 has balanced economic stimulus with fiscal discipline. The
stimulus comes from a 33% increase in public investments covering roads, ports, airports and
railways. The income tax rejig may also produce higher demand if people divert their tax
saving towards spending.

Therefore, option (a) is the correct answer.


Relevance: Recently, the Union Budget 2024 has announced a 33% increase in public investments.

Question 18. + 2.0 - 0.66

Consider the following statements:

Statement-I:
The bond prices tend to increase with decrease in the interest rates by the banks.

Statement-II:

Most Bonds pay a fixed interest rate that provides higher returns than the bank deposits.
Which one of the following is correct in respect of the above statements?

Both Statement-I and Statement-II are correct and Statement-II is the correct
 A.
explanation for Statement-I

Both Statement-I and Statement-II are correct and Statement-II is not the correct
 B.
explanation for Statement-I

C. Statement-I is correct but Statement-II is incorrect

D. Statement-I Is incorrect but Statement-II is correct

Question Analytics
4303 USERS 1980 USERS 2323 USERS 361.1 SECS

ATTEMPTED  SOLVED CORRECTLY  SOLVED INCORRECTLY  YOUR TIME

70.31 SECS 0.0 SECS

 AVG. SOLVING TIME  FASTEST SOLVING TIME

Explanation :

Bonds have an inverse relationship to interest rates. When the cost of borrowing money
rises (when interest rates rise), bond prices usually fall, and vice-versa. So, statement 1 is
correct.
Most bonds pay a fixed interest rate that becomes more attractive if interest rates fall,
driving up demand and the price of the bond. So, statement 2 is correct.
As interest rates decrease, the present value of the fixed future cash flows from a bond
increases. This is because the discount rate used to calculate present value (which is based on
prevailing interest rates) is lower, leading to a higher present value.
A higher present value translates to a higher market price for the bond. Conversely, if interest
rates rise, investors will no longer prefer the lower fixed interest rate paid by a bond, resulting
in a decline in its price.
Zero-coupon bonds provide a clear example of how this mechanism works in practice.

So, both Statement-I and Statement-II are correct and Statement-II is the correct explanation
for Statement-I.

Therefore, option (a) is the correct answer.


Relevance: Recently, Bloomberg data showed that the 10-year benchmark bond yield closed at
7.01%, its lowest level since June 14, 2023.

Question 19. + 2.0 - 0.66

Consider the following statements with reference to the Total Expense Ratio (TER):

1. It is a measure of the operational efficiency of mutual funds.


2. It is used to calculate the return on investments after consideration of all fees.

3. The lower the expense ratio, the lower the returns on investment.

How many of the above statements are correct?

A. Only one

 B. Only two

C. All three

D. None

Question Analytics
2661 USERS 2015 USERS 646 USERS 15.6 SECS

ATTEMPTED  SOLVED CORRECTLY  SOLVED INCORRECTLY  YOUR TIME

39.75 SECS 0.0 SECS

 AVG. SOLVING TIME  FASTEST SOLVING TIME

Explanation :
Total Expense Ratio (TER):

It is a measure of the total costs associated with managing and operating an investment
fund, such as a mutual fund. It is also known as the net expense ratio or after-
reimbursement expense ratio. The total cost of the fund is divided by the fund’s total assets
to arrive at a percentage amount, which represents the TER. It throws light on the operational
efficiency of mutual funds. So, statement 1 is correct.
The TER covers various expenses, including management fees, administrative fees,
distribution fees, auditing fees, legal fees, and other operating expenses. It helps investors
understand the overall cost they will bear when investing in a mutual fund. The TER
directly affects the overall return from the mutual fund. Higher TER means higher expenses.
For Example: For any scheme, if its return is 22% and its expense ratio is 2.25%, then the net
return for the investor will be 19.75%. So, statement 2 is correct.
It is used by investors to compare the costs of the scheme with its peers and also about the
returns available from that scheme. It is a key element in making an investment choice. A
lower TER could mean a higher profitability rate. Those funds which consistently show a
high TER may not provide high returns, since high expenses tend to erode the returns
generated. So, statement 3 is not correct.

So, only two of the above statements are correct.

Therefore, option (b) is the correct answer.


Relevance: The Securities and Exchange Board of India has recently decided to issue a fresh
consultation on the issue of Total Expense Ratio (TER) for mutual funds.

Question 20. + 2.0 - 0.66

An increase in the interest rate in an economy will most likely lead to:

A. increase in investment by businesses

 B. higher individual savings

C. increase in consumer spending

 D. higher inflation rates

Question Analytics
4889 USERS 3547 USERS 1342 USERS 34.5 SECS
ATTEMPTED  SOLVED CORRECTLY  SOLVED INCORRECTLY  YOUR TIME

50.32 SECS 0.0 SECS

 AVG. SOLVING TIME  FASTEST SOLVING TIME

Explanation :

Higher savings usually occur when interest rates are higher, as individuals seek better
returns on their savings in interest-bearing accounts.
On the contrary, lower interest rates make borrowing cheaper which encourages businesses
to invest in expansion and individuals to take loans for various purposes.
When interest rates are lowered, it tends to encourage individuals to take out loans for major
purchases such as homes or cars, leading to increased spending. This contributes to increased
money supply in the economy, which can put upward pressure on prices rather than
reducing inflation.
Businesses, with reduced costs of financing, are incentivized to invest in new projects and
initiatives. This contributes to overall economic growth.

Therefore, option (b) is the correct answer.


Relevance: The Reserve Bank of India’s Monetary Policy Committee has decided to keep the policy
repo rate unchanged at 6.5% as retail inflation continues to be above its target of 4%.

Question 21. + 2.0 - 0.66

Consider the following statements:

1. Hibernation allows animals to increase metabolism during periods of high temperatures.


2. Aestivation is characterised by lower body temperatures to survive long cold winters.

3. Brumation is the state of dormancy observed in cold-blooded animals during cold seasons.
How many of the above statements are correct?

 A. Only one

 B. Only two

C. All three

D. None

Question Analytics
3806 USERS 2148 USERS 1658 USERS 88.5 SECS
ATTEMPTED  SOLVED CORRECTLY  SOLVED INCORRECTLY  YOUR TIME

48.68 SECS 0.0 SECS

 AVG. SOLVING TIME  FASTEST SOLVING TIME

Explanation :

Hibernation is a prolonged state of torpor that helps animals to slow down their heart rate
and lower their body temperature for extended periods to survive long cold winters.
Many rodent’s body temperatures reach almost freezing at this time. The majority of these
hibernating animals still have a food store, waking up every few weeks to eat something and
return to hibernate again until they can wake up properly in spring. So, statement 1 is not
correct.
Aestivation is a prolonged state of torpor that animals undergo in the heat or during
drought. The mechanism is similar to hibernation, where organisms use their physiology to
slow their metabolism way down and enter a deep sleep, reducing the need for food and
especially water. This happens in places like deserts but has also been found in marine
habitats, where marine animals go more dormant as water temperature is higher and food
availability is low. So, statement 2 is not correct.
Brumation is the state of dormancy or slowed metabolic activity observed in certain
ectothermic (cold-blooded) animals, particularly reptiles, during colder months. It is also a
form of prolonged hypometabolism, but brumation is not torpor because it is a type of
hypometabolism undergone by ectothermic animals. These ectotherms simply allow their
environment to cool their body down and slow down their metabolism. Common examples
include snakes, lizards, turtles and certain types of frogs. So, statement 3 is correct.

So, only one of the above statements is correct.


Therefore, option (a) is the correct answer.

Knowledge Box

Torpor or cold-lethargy is a hypometabolic state (very low metabolism) associated with low
body temperature. It is used by endothermic animals to save energy to survive harsh
conditions such as cold temperatures or lack of food.
Daily torpor is distinct from hibernation and aestivation, which are considered prolonged
torpor.
Hummingbirds are a great example of daily torpor in animals, going into a deep sleep state
every single night. Animals undergoing daily torpor have an extremely high metabolism and
need to feed every few minutes, so at night, they reduce their heart rate and body temperature
to rest properly. It can also apply to animals that in general have poor diets, such as koala
bears and sloths. Animals in deserts like cactus mice (Peromyscus eremicus) undergo daily
torpor, to avoid extreme heat during the day.

Relevance: Experts suggest that hibernation may hold important clues on how to slow down ageing
processes.

Question 22. + 2.0 - 0.66

Consider the following:


1. Methane

2. Nitrous oxide
3. Chlorofluorocarbons

4. Ozone

5. Sulfur hexafluoride
How many of the above greenhouse gases occur naturally in the atmosphere?

A. Only two

  B. Only three

C. Only four

D. All five

Question Analytics
4768 USERS 2667 USERS 2101 USERS 25.6 SECS

ATTEMPTED  SOLVED CORRECTLY  SOLVED INCORRECTLY  YOUR TIME

44.29 SECS 0.0 SECS

 AVG. SOLVING TIME  FASTEST SOLVING TIME

Explanation :

Greenhouse gas refers to any gas that has the property of absorbing infrared radiation emitted
from the Earth’s surface and re-radiating it back to the Earth’s surface leading to an increase in
temperature, thus contributing to the greenhouse effect.

Some Greenhouse Gases (GHGs) occur naturally in the atmosphere, while others result from
human activities. Naturally occurring greenhouse gases include:
Water Vapour
Carbon dioxide
Methane. So, point 1 is correct.
Nitrous oxide. So, point 2 is correct
Ozone. So, point 4 is correct.
Many synthetic, powerful greenhouse gases are emitted from a variety of household,
commercial, and industrial applications and processes. Some of them are:
Hydrofluorocarbons
Perfluorocarbons
Nitrogen trifluoride
Chlorofluorocarbons. So, point 3 is not correct.
Sulfur hexafluoride. So, point 5 is not correct.

So, only three of the above greenhouse gases occur naturally in the atmosphere.
Therefore, option (b) is the correct answer.

Relevance: As per United Nations’s World Meteorological Organisation, greenhouse gases have hit
record high in 2023.

Question 23. + 2.0 - 0.66

In India, no alteration of the boundaries of a National Park by the State Government can be made
except on a recommendation of:

  A. National Board for Wildlife (NBWL)

B. National Green Tribunal (NGT)

C. National Biodiversity Authority (NBA)

D. Animal Welfare Board of India (AWBI)

Question Analytics
4708 USERS 3317 USERS 1391 USERS 29.0 SECS

ATTEMPTED  SOLVED CORRECTLY  SOLVED INCORRECTLY  YOUR TIME

31.53 SECS 0.0 SECS


31.53 SECS 0.0 SECS

 AVG. SOLVING TIME  FASTEST SOLVING TIME

Explanation :

In India, the Wildlife Protection Act, 1972 is a key legislation that governs the conservation
and protection of wildlife and their habitats. Under this Act, the establishment, management
and regulation of National Parks fall under the purview of the State Government.
To ensure that such decisions related to any alteration to the boundaries of a National Park are
made with due diligence and expertise, the above Act mandates that any alteration to the
boundaries of a National Park must be recommended by the National Board for Wildlife
(NBWL).
The NBWL is a statutory organisation constituted under the Wildlife Protection Act,
1972, chaired by the Prime Minister of India and comprises eminent conservationists,
scientists and government officials.
By involving the NBWL in the decision-making process, there is an added layer of scrutiny
and evaluation to ensure that changes to National Park boundaries are in line with conservation
objectives to minimise adverse impacts on the environment and wildlife.

Therefore, option (a) is the correct answer.

Relevance: The National Board for Wildlife has accepted two elevated railway lines surrounding
Deepor Beel lake in Assam.

Question 24. + 2.0 - 0.66

Consider the following statements:


1. Global Tiger Initiative (GTI) was launched by the World Bank and Global Environment Facility.

2. Global Tiger Forum (GTF) is an intergovernmental body including all tiger range countries.

3. Conservation Assured Tiger Standards (CA|TS) is a global tool that sets minimum standards for
management of tigers.
4. Only 14 Tiger Reserves in India have received the CA|TS accreditation.

How many of the above statements are correct?

A. Only one

 B. Only two

 C. Only three
D. All four

Question Analytics
2743 USERS 1237 USERS 1506 USERS 27.1 SECS

ATTEMPTED  SOLVED CORRECTLY  SOLVED INCORRECTLY  YOUR TIME

40.51 SECS 0.0 SECS

 AVG. SOLVING TIME  FASTEST SOLVING TIME

Explanation :

The Global Tiger Initiative (GTI) was launched by the World Bank, Global Environment
Facility, Smithsonian Institution, Save the Tiger Fund, and International Tiger Coalition.
The GTI is led by the 13 tiger range countries. It is a global alliance of governments,
international organisations, civil society, the conservation and scientific community and the
private sector that are committed to working together toward a common agenda to save wild
tigers from extinction. So, statement 1 is correct.
The Global Tiger Forum (GTF) is an international intergovernmental body exclusively set
up for the conservation of tigers in the wild in certain tiger range countries. Out of the 13 tiger
range countries, seven are currently members of GTF: Bangladesh, Bhutan, Cambodia,
India, Myanmar, Nepal and Vietnam besides non-tiger range country, the United Kingdom.
The secretariat is based in New Delhi, India. So, statement 2 is not correct.
Conservation Assured Tiger Standards (CA|TS) is a globally accepted conservation tool
that sets best practices and standards to manage tigers and encourages assessments to
benchmark progress. It sets minimum standards for the effective management of target species
and encourages the assessment of these standards. It is being implemented in seven tiger range
countries. So, statement 3 is correct.
So far a total of 23 tiger reserves of India have received CA|TS accreditation.
14 Tiger Reserves in India have received the accreditation of the CA|TS in 2021:
Manas, Kaziranga and Orang in Assam, Satpura, Kanha and Panna in Madhya Pradesh,
Pench in Maharashtra, Valmiki in Bihar, Dudhwa in Uttar Pradesh, Sunderbans in West
Bengal, Parambikulam in Kerala, Bandipur in Karnataka and Mudumalai and Anamalai
in Tamil Nadu.
3 tiger reserves Sathyamangalam in Tamil Nadu and Bandhavgarh and Pench in
Madhya Pradesh have been conferred with the Conservation Assured Tiger Standards
(CA|TS) in 2022.
6 tiger reserves namely Kali, Melghat, Navegaon, Nagzira, Pilibhit and Periyar have
been awarded with CA|TS accreditation in 2023. So, statement 4 is not correct.

So, only two of the above statements are correct.


Therefore, option (b) is the correct answer.

Knowledge Box

Project Tiger:

It was launched in 1973 by the Government of India. It aims to foster an exclusive tiger
agenda in the core areas of tiger reserves, with an inclusive people-oriented agenda in the
buffer zone.
It is an ongoing Centrally Sponsored Scheme of the Union Ministry of Environment, Forests
and Climate Change providing central assistance to the tiger States for their conservation in
designated tiger reserves.

Relevance: The National Tiger Conservation Authority (NTCA) recently approved shifting three
tigresses from Ranthambore Tiger Reserve (RTR) to two other State reserves.

Question 25. + 2.0 - 0.66

The ‘Cut Rootstock Method’ is sometimes seen in the news for which one of the following purposes?

A. Clearing off the agricultural field after harvest in a sustainable manner

B. Increasing water retention capacity in paddy cultivation

C. Eliminating the need for soil preparation in vertical farming

 D. Protecting plants from invasive weed species

Question Analytics
3719 USERS 967 USERS 2752 USERS 0.2 SECS

ATTEMPTED  SOLVED CORRECTLY  SOLVED INCORRECTLY  YOUR TIME

49.48 SECS 0.0 SECS

 AVG. SOLVING TIME  FASTEST SOLVING TIME

Explanation :

Cut Rootstock Method:


It is a scientific approach to controlling the spread of Lantana Camara, an invasive weed
species and to make way for native plants. The weed has spread over a large part hence,
obstructing the way for native plants.
The method attempts to make the growth of weed dormant through a hormonal imbalance and
ensures it doesn’t germinate. It is done by manually uprooting the weed and cutting its roots
underneath the ground by an edge of a few inches. The cycle additionally includes holding
the evacuated plant upside down to kill it and removing the more modest lantana saplings,
alongside the greater weed, physically.
When the species is cleared, it permits the proliferation of the local bushes and creepers,
which have frequently been viewed as immature or are attempting to develop normally close to
the invasive species. This method of weeding does not require any automated machine.

Therefore, option (d) is the correct answer.

Relevance: Recently, the Cut Rootstock Method was suggested to control the spread of Lantana
Camara in Asola Bhatti Wildlife Sanctuary.

Question 26. + 2.0 - 0.66

Which one of the following was recently declared as India’s first Dark Sky Park?

A. Changthang Wildlife Sanctuary in Ladakh

B. Desert National Park in Rajasthan

C. Valley of Flowers National Park in Uttarakhand

 D. Pench Tiger Reserve in Maharashtra

Question Analytics
4362 USERS 1493 USERS 2869 USERS 46.7 SECS

ATTEMPTED  SOLVED CORRECTLY  SOLVED INCORRECTLY  YOUR TIME

22.95 SECS 0.0 SECS

 AVG. SOLVING TIME  FASTEST SOLVING TIME

Explanation :

The Pench Tiger Reserve (PTR) in Maharashtra has attained the distinction of becoming
India’s inaugural Dark Sky Park and the fifth in Asia.
The PTR, also known as Pench National Park, stands as one of India’s premier Tiger
Reserves and the first to span two States – Madhya Pradesh and Maharashtra.
The decision to designate Pench as the first Dark Sky Park is driven by the need to
shield the night sky from light pollution.
India’s first dark-sky Reserve (not Park) is the Indian Astronomical Observatory (IAO),
a high-altitude astronomy station situated in Hanle village (part of Changthang WLS) and
operated by the Indian Institute of Astrophysics.
Positioned in the Western Himalayas at an elevation of 4,500 meters (14,764 ft), the IAO
ranks among the world’s highest sites for optical, infrared and gamma-ray telescopes.

Therefore, option (d) is the correct answer.

Knowledge Box

Dark Sky Park:

They are areas designated for the protection of night skies from light pollution, resulting in an
environment that allows for the local ecosystem to thrive.
These places are not only important for nature and wildlife but equally aid in research for
those interested in astronomy.
The use of artificial lights to provide visibility affects humans, animals and hinders the ability
to gaze into the celestial landscape with clarity.
A dark-sky preserve is an area, typically surrounding a park or observatory, that limits
artificial light pollution. The primary objective of the dark-sky movement is generally to
endorse astronomy.
Difference between Dark Sky Reserves and Dark Sky Park:
Reserves consist of a core area meeting minimum criteria for sky quality and natural
darkness, and a peripheral area that supports dark sky preservation in the core.
Reserves are formed through a partnership of multiple land managers who have
recognized the value of the natural nighttime environment through regulations and
long-term planning.
The land may be publicly or privately owned, provided that the landowner(s) consent
to the right of permanent, ongoing public access to specific areas included in the
DarkSky Park designation.

Relevance: Recently, Pench Tiger Reserve became India’s first Dark Sky Park.
Question 27. + 2.0 - 0.66

Which of the following are marine mammals?

1. Seals

2. Whales
3. Walruses

4. Penguins
5. Dugongs

Select the correct answer using the code given below:

A. 1, 2 and 4 only

B. 3, 4 and 5 only

 C. 1, 2, 3 and 5 only

 D. 1, 2, 3, 4 and 5

Question Analytics
4642 USERS 3036 USERS 1606 USERS 37.9 SECS

ATTEMPTED  SOLVED CORRECTLY  SOLVED INCORRECTLY  YOUR TIME

44.09 SECS 0.0 SECS

 AVG. SOLVING TIME  FASTEST SOLVING TIME

Explanation :

Marine mammals are found in marine ecosystems around the globe. They are a diverse group
of mammals with unique physical adaptations that allow them to thrive in the marine
environment with extreme temperatures, depths, pressure, and darkness.
Marine mammals are classified into four different taxonomic groups:
Cetaceans: whales, dolphins, and porpoises etc.
Pinnipeds: seals, sea lions and walruses etc.
Sirenians: manatees and dugongs etc.
Marine fissipeds: polar bears and sea otters etc.
So, seals, whales, walruses and dugongs are marine mammals. So, points 1, 2, 3 and
5 are correct.
Penguins are specialised marine birds (not marine mammals) adapted to living at sea. So,
point 4 is not correct.

Therefore, option (c) is the correct answer.

Relevance: According to the IUCN Red List human activity and climate change is devastating
marine species from mammals to corals.

Question 28. + 2.0 - 0.66

In India, hunting of a wild animal mentioned in Schedule I of the Wildlife Protection Act, 1972, is
permitted in which of the following circumstances?

1. If a wild animal is causing damage to property.


2. If a wild animal is disabled or diseased beyond recovery and cannot be captured.

3. If a person was acting in defense of oneself or another person.

Select the correct answer using the code given below:

A. 1 only

 B. 1 and 3 only

C. 2 only

 D. 2 and 3 only

Question Analytics
4607 USERS 2516 USERS 2091 USERS 44.0 SECS

ATTEMPTED  SOLVED CORRECTLY  SOLVED INCORRECTLY  YOUR TIME

55.14 SECS 0.0 SECS

 AVG. SOLVING TIME  FASTEST SOLVING TIME

Explanation :

Hunting of wild animals specified in Schedule I of the Wildlife Protection Act, 1972 can only
be permitted when they become dangerous to human life or are beyond recovery, not when
causing damage to property. So, statement 1 is not correct.
The Chief Wildlife Warden may permit the hunting of a wild animal specified in Schedule I if
it is disabled or diseased beyond recovery and cannot be captured, tranquilised or
translocated. So, statement 2 is correct.
While hunting is allowed in self-defense or in defense of another person, it should be noted
that this should not exonerate anyone who was committing an act in contravention of the
Wildlife Protection Act at the time when such defense was necessary. So, statement 3 is
correct.
The Chief Wildlife Warden can grant a permit for hunting wild animals for education,
scientific research, or scientific management purposes, provided certain conditions are met.

Therefore, option (d) is the correct answer.

Relevance: The state of Kerala urged the Union Government to make some amendments to the
Wildlife Protection Act of 1972.

Question 29. + 2.0 - 0.66

Terracotta models of mummies were discovered at which one of the following Indus Valley
Civilization sites?

 A. Rakhigarhi

B. Dholavira

C. Banawali

 D. Lothal

Question Analytics
3521 USERS 1120 USERS 2401 USERS 17.8 SECS

ATTEMPTED  SOLVED CORRECTLY  SOLVED INCORRECTLY  YOUR TIME

29.13 SECS 0.0 SECS

 AVG. SOLVING TIME  FASTEST SOLVING TIME

Explanation :

Lothal is one of the prominent sites of the ancient Harappan Civilization, dating to 2600 BCE
to 2000 BCE. It is located close to the Gulf of Khambhat.
The artefacts found in Lothal include models of boats, Persian Gulf seals, bangles, a model of
a terracotta mummy, a terracotta pyramid, a Sumerian head, precious and semi-precious
stones like carnelian and lapis lazuli.
Being a port town, Lothal hosted traders from different parts of the world and civilizations.
Along with trade products, their cultural beliefs and values, ideas and innovative techniques
also transferred to Lothal, making it the hub of the culture and economy of the world.

Therefore, option (d) is the correct answer.

Question 30. + 2.0 - 0.66

Consider the following:

1. Establishment of the institution of gotra


2. Nuclearisation of families

3. Beginning of the usage of iron

4. Emergence of temple worship


How many of the above developments took place during the Later Vedic phase?

A. Only one

  B. Only two

C. Only three

D. All four

Question Analytics
4233 USERS 1764 USERS 2469 USERS 44.5 SECS

ATTEMPTED  SOLVED CORRECTLY  SOLVED INCORRECTLY  YOUR TIME

48.23 SECS 0.0 SECS

 AVG. SOLVING TIME  FASTEST SOLVING TIME

Explanation :

Later Vedic Phase (1000 BC to 500 BC):

The institution of gotra developed in this period. This means that people having a common
gotra descended from a common ancestor and no marriage between the members of the
same gotra could take place. Monogamous marriages were preferred even though polygamy
was frequent. Women had to stay with their husbands at their place after marriage. So, point 1
is correct.
The family remained the basic unit of the Vedic society. However, its composition changed.
The later Vedic family became large enough to be called a joint family with three or four
generations living together. The rows of hearths discovered at Atranjikhera and at
Ahichchhatra (both in western Uttar Pradesh) show that these were meant for communal
feeding or for cooking the food of large families. So, point 2 is not correct.
The main factor in the expansion of the Aryan culture during the later Vedic period was the
beginning of the use of iron around 1000 BC. The Rigvedic people knew of a metal called
ayas which was either copper or bronze. In the later Vedic literature ayas were qualified with
Shyama or Krishna meaning black to denote iron. So, point 3 is correct.
During the entire Vedic phase, people did not construct temples nor did they worship any
statues. These features of Indian religion developed much later. So, point 4 is not correct.
There was an increase in the frequency and number of the yajna which generally ended with
the sacrifices of a large number of animals. This was probably the result of the growing
importance of a class of Brahmanas and their efforts to maintain their supremacy in the
changing society. Some of the important yajnas were - ashvamedha, vajapeya, rajasuya etc.

So, only two of the above developments took place during the Later Vedic phase.
Therefore, option (b) is the correct answer.

Question 31. + 2.0 - 0.66

Consider the following Ashokan inscriptions:

1. Laghman
2. Ahraura

3. Mansehra
4. Shahbazgarhi

How many of the above are not found in India?

A. Only one

B. Only two

 C. Only three

D. All four

Question Analytics
1995 USERS 589 USERS 1406 USERS 16.1 SECS

ATTEMPTED  SOLVED CORRECTLY  SOLVED INCORRECTLY  YOUR TIME

24.02 SECS 0.0 SECS


 AVG. SOLVING TIME  FASTEST SOLVING TIME

Explanation :
Ashoka (268 B.C.E to 232 B.C.E.) issued a large number of edicts that carried his message
concerning the idea and practice of dhamma. The Ashokan inscriptions were written in
multiple languages. The northwestern part of the Mauryan Empire (which included areas of
modern-day Pakistan and Afghanistan) yielded one Greek, four Aramaic, one Greek-Aramaic
and one Aramaic-Prakrit inscription.
An inscription is found in the Laghman District on the left (northern) bank of the Kabul
River, a little above Jalalabad, in modern Afghanistan. So, point 1 is correct.
A minor inscription is found in the Ahraura, which is located in the Mirzapur district of Uttar
Pradesh. So, point 2 is not correct.
Mansehra and Shahbazgarhi inscriptions are located in Pakistan. They are sets of Major
Rock Edicts in the Prakrit language and Kharosthi script. So, points 3 and 4 are correct.
The fourteen major edicts recorded at Shahbazgarhi site present aspects of Asoka’s
dharma or righteous law. The edicts are located beside one of the ancient trade routes
connecting the Vale of Peshawar with the valley of Swat, Dir and Chitral to the North
and the great city of Taxila to the South East.

So, only three of the above inscriptions are not found in India.

Therefore, option (c) is the correct answer.

Question 32. + 2.0 - 0.66

The notion of Arhat in Jainism and Bodhisattva in Buddhism is similar in which one of the following
ways?

A. Both seek enlightenment for themselves alone, ignoring the well-being of others.

B. Both believe that there is only a single facet of ultimate truth.

  C. Both seek to break away from the cycle of death and rebirth.

D. Both do not believe that actions can help to accumulate good karma.

Question Analytics
4508 USERS 3030 USERS 1478 USERS 83.3 SECS

ATTEMPTED  SOLVED CORRECTLY  SOLVED INCORRECTLY  YOUR TIME

50.45 SECS 0.0 SECS

 AVG. SOLVING TIME  FASTEST SOLVING TIME


Explanation :

While Arhats in Jainism aim for their own enlightenment, Bodhisattvas in Buddhism
prioritise the well-being and enlightenment of all sentient beings before achieving it
themselves. While Arhats in Jainism adhere to a strict set of rules, Bodhisattvas in Buddhism
follow a more flexible set of guidelines known as the "six paramitas".
Both Arhats in Jainism and Bodhisattvas in Buddhism seek liberation from the cycle of death
and rebirth.
While both Arhats and Bodhisattvas aim to accumulate positive karma through their
actions, their primary goal is not merely to accumulate good karma but to attain liberation
from the cycle of rebirth.
Anekantavada is the fundamental doctrine of Jainism that emphasises that the ultimate
truth and reality are complex and have multiple aspects. Hence, there exists non-absolutism,
which means no single, specific statement can describe the nature of existence and the absolute
truth.

Therefore, option (c) is the correct answer.

Knowledge Box

About Arhat:

In the Jain sūtras, the term is often used in a sense closer to that found in Buddhist writings.
Here the arhat is described as one who is free from desire, hatred and delusion, who
knows everything, and who is endowed with miraculous powers.
While these characterizations are consistent with the Buddhist use of the term, it should be
noted that the Jains applied the word exclusively to the tīrthankaras or revealers of
religion, whereas in Buddhism arhatship is an ideal to be attained by all serious religious
strivers, especially monks and nuns.

Question 33. + 2.0 - 0.66

With reference to the Delhi Sultanate, consider the following statements:


1. Alauddin Khilji used taxes from khalisa land for payment of cash salaries to the soldiers.

2. Muhammad bin Tughlaq extended Alauddin Khilji’s system of revenue collection based on
measurement of land.

3. Firoz Shah Tughlaq introduced the levy of house tax (ghari) and cattle tax (charai).
Which of the statements given above are correct?

 A. 1 and 2 only

B. 2 and 3 only

C. 1 and 3 only

 D. 1, 2 and 3

Question Analytics
3065 USERS 1239 USERS 1826 USERS 20.0 SECS

ATTEMPTED  SOLVED CORRECTLY  SOLVED INCORRECTLY  YOUR TIME

41.54 SECS 0.0 SECS

 AVG. SOLVING TIME  FASTEST SOLVING TIME

Explanation :

The rulers who ruled substantial parts of North India between AD1200 to AD1526 were termed
Sultans and the period of their rule was the Delhi Sultanate. These rulers were of Turkish and Afghan
origin. During this period, five different dynasties ruled Delhi: Mamluks (AD 1206–AD 1290)
(popularly known as the slave dynasty), the Khiljis (AD 1290–AD 1320), the Tughlaqs (AD 1320–
AD 1412), the Sayyids (AD 1412–AD 1451) and the Lodis (AD 1451– AD 1526).

Alauddin Khilji attempted to increase the revenue collection by enhancing the demand,
introducing direct collection and cutting down the leakages to the intermediaries. The revenue
collectors were ordered to demand the revenue with such rigour that the peasants should be
forced to sell their produce immediately at the side of the fields. At another place, Alauddin
Khalji brought the Doab into Khalisa and the tax (mahsul) from there was spent on paying
the cash salaries to the soldiers. So, statement 1 is correct.
Muhammad bin Tughlaq first extended Alauddin Khilji’s system of revenue collection
based on measurement of land to Gujarat, Malwa, Deccan, South India and Bengal. There was
an enormous increase in agrarian taxation. Some additional new imposts (abwab) were levied.
Of the other taxes, kharaj, charai and ghari were more rigorously collected. He introduced the
practice of giving agricultural loans named sondhar for increasing the area under plough
and for digging wells for irrigation. A new ministry designated diwan-i amir-i kohi was
established to promote agriculture. He became the first Sultan to attempt to formulate an
agricultural policy for promoting agriculture. So, statement 2 is correct.
Firuz Shah Tughlaq (1351-88) abolished agrarian cesses and forbade levying of ghari
(house tax) and charai (cattle tax). But he is reported to have imposed a separate tax – jiziya
– distinct from kharaj (land tax) on the peasants. He also introduced an irrigation tax (haqq-i
sharb) in Haryana where he dug up canals. So, statement 3 is not correct.

Therefore, option (a) is the correct answer.

Knowledge Box

Under Firuz Shah, ‘water tax’ (haqq-i sharb) was taken from those cultivators who
irrigated their land from the water supplied by the canals constructed by the state.
Khalisa: It referred to territories whose revenues were directly collected for the Sultan’s own
treasury. Its size seems to have expanded quite considerably under Alauddin Khalji. However,
it did not appear to consist of shifting territories scattered throughout the country.

Question 34. + 2.0 - 0.66

Which one of the following best describes the terms ‘Theragatha’ and ‘Therigatha’?

A. Kirtanghar established by Sankardeva

B. Jaina temples in Southern India

C. Hymns composed by Alvar saints

  D. Collection of poems narrated by Buddhist monks and nuns

Question Analytics
4151 USERS 3315 USERS 836 USERS 61.0 SECS

ATTEMPTED  SOLVED CORRECTLY  SOLVED INCORRECTLY  YOUR TIME

27.25 SECS 0.0 SECS

 AVG. SOLVING TIME  FASTEST SOLVING TIME

Explanation :

Theragatha (Verses of Elder Monks) and Therigatha (Verses of the Elder Nuns) are a
collection of poems with verses which were narrated by the early members of the Buddhist
Sangha.
In Theravada Buddhism, the Sutta Pitaka is a compilation of discourses ascribed to the
Buddha.
Specifically, the Khuddaka Nikaya holds a unique position within this collection,
encompassing diverse literary genres such as verses on ethics (Dhammapada), hymns by
monks (Theragatha), hymns by nuns (Therigatha), and narrative tales illustrating moral lessons
(Jataka).
Therigatha is the first surviving poetry supposed to have been composed by women in
India. Hence, it is important for not only Buddhism but also gender studies. The gathas of the
Therigatha strongly support the view that women are equal to men in terms of spiritual
attainment.

Therefore, option (d) is the correct answer.

Question 35. + 2.0 - 0.66

The motto of the Supreme Court of India, 'yatodharmastato jayah' (Truth alone I uphold) is taken
from:

 A. Mundaka Upanishad

 B. Mahabharata

C. Ramayana

D. Yajurveda

Question Analytics
4347 USERS 2245 USERS 2102 USERS 83.8 SECS

ATTEMPTED  SOLVED CORRECTLY  SOLVED INCORRECTLY  YOUR TIME

29.81 SECS 0.0 SECS

 AVG. SOLVING TIME  FASTEST SOLVING TIME

Explanation :
The inscription on the Dharm Chakra logo of the Supreme Court in Sanskrit
“yatodharmastato jayah” means – Truth alone I uphold. It is also referred to as the wheel
of righteousness, encompassing truth, goodness and equity.
It is a sober reminder to those who judge the nation’s laws against the high standards of the
Constitution, of the high standards to which they too must hold themselves.
The phrase comes from the Hindu epic Mahabharata verse 13.153.39.

Therefore, option (b) is the correct answer.

Question 36. + 2.0 - 0.66

Consider the following pairs:

Sl. No. Temple Architecture Style Example

1. Solanki : Somnath Temple at Kathiawar

2. Nagara : Kailashnath Temple at Ellora

3. Vesara : Doddabasappa Temple at Dambal

4. Dravidian : Chennakeshava Temple at Belur

How many of the above pairs are correctly matched?

A. Only one pair


 B. Only two pairs

 C. Only three pairs

D. All four pairs

Question Analytics
2974 USERS 1600 USERS 1374 USERS 96.8 SECS

ATTEMPTED  SOLVED CORRECTLY  SOLVED INCORRECTLY  YOUR TIME

41.55 SECS 0.0 SECS

 AVG. SOLVING TIME  FASTEST SOLVING TIME

Explanation :

Solanki Style of Temple Architecture: It flourished in Gujarat. It consists of a closed hall


(‘sanctum’) and a porch that is inter-connected both internally and externally. A detached peri-
stylar hall is added in larger temples on the same axis, which is often preceded by a ‘torana’ or
ornamental arched entrance. These temples were largely built in sandstone or limestone. The
Sun Temple at Modhera, the Vemala Temple at Mount Abu and the Somnath Temple at
Kathiawar are examples of the Solanki architecture. So, pair 1 is correctly matched.
Nagara Style of Temple Architecture: It emerged in northern India in the 5th century CE,
during the late Gupta period. The plan of this style is four-sided. The garbhagriha is a
perfect square whereas the whole temple plan could be oblong. The garbhagriha or sanctum
houses the image or idol of the main deity. The pathway leading to the garbhagriha is the
mandapa. The shikhara is tapered at the top. Garbhagriha is surrounded by a
pradakshinapatha or a circumambulatory path. It generally rests on a high platform.
Examples of Nagara Temples include the Khajuraho Temple. So, pair 2 is not correctly
matched.
Vesara Style of Temple Architecture: It is also known as the Karnataka School of
Architecture. It was conceptualised under the later Chalukya rulers in the mid-seventh
century CE. It combined the features of both Nagara school and Dravidian school and
resulted in a hybridised style. Some of its features emphasise vimana and mandapa, an open
ambulatory passageway. The pillars, doorways and ceilings were decorated with intricate
carvings. Examples include the Kailashnath temple in Ellora, Doddabasappa Temple at
Dambal. So, pair 3 is correctly matched.
Dravidian Style of Temple Architecture: It flourished under the patronage of the Chola
rulers. It was a continuation of the previous Pallava architecture, with some variations. Unlike
the Nagara temples, the Dravidian temples were surrounded by high boundary walls. The
front wall had a high entrance gateway known as the gopuram. The temple premise was laid
out in the panchayatan style with a principal temple and four subsidiary shrines.
In the Dravidian style, the spire is in the form of a stepped pyramid that rises up linearly
rather than curved. It is known as vimana. There is only one vimana in the Dravidian
architecture on top of the main temple. The subsidiary shrines do not have vimanas,
unlike in Nagara architecture. The presence of a water tank inside the temple enclosure
is a unique feature of the Dravidian style. Examples include Brihadeswara temple at
Tanjore and Gangaikondacholapuram temple.
Hoysala Style of Temple Architecture: In the region of Karnataka (near Mysore), the temples
built under the Hoysala rulers developed a distinct style of their own known as the Hoysala
School of art. Unlike the crucified ground plan of the Panchayatan style, the shrines led out in
the shape of an intricately designed star. This was known as the Stellate plan. Examples
include Hoysaleswara Temple at Halebidu, Chennakesava Temple at Belur, Chennakesava
Temple at Somanathapura. So, pair 4 is not correctly matched.

So, only two of the above pairs are correctly matched.

Therefore, option (b) is the correct answer.


Relevance: Recently, three Hoysala-era temples in Karnataka were listed in UNESCO’s World
Heritage List.

Question 37. + 2.0 - 0.66

In the context of the social reforms during British India, which one of the following organisations
was formed earliest?

A. Satyashodhak Samaj

  B. Prarthana Samaj

C. Arya Samaj

D. Ramakrishna Mission

Question Analytics
4329 USERS 2078 USERS 2251 USERS 80.0 SECS

ATTEMPTED  SOLVED CORRECTLY  SOLVED INCORRECTLY  YOUR TIME

37.85 SECS 0.0 SECS

 AVG. SOLVING TIME  FASTEST SOLVING TIME

Explanation :

Prarthana Samaj: In 1867, Keshab Chandra Sen helped Atmaram Pandurang found the
Prarthana Samaj in Bombay. Earlier, the Brahmo ideas spread in Maharashtra. A precursor of
the Prarthana Samaj was the Paramahansa Sabha, something like a secret society to spread
liberal ideas and encourage the breakdown of caste and communal barriers. The emphasis was
on monotheism, but on the whole, the samaj was more concerned with social reforms than with
religion.
Satyashodhak Samaj (Society of Seekers of Truth): Jyotiba Phule formed this in 1873 to
educate society about caste prejudice and to free downtrodden lower-caste people from the
stigmas created by Brahmins. Its main objectives included social service and the spread of
education among women and lower caste people.
Arya Samaj: It was founded by Swami Dayanand Saraswati in 1875 in Bombay. The
important task of this Socio-Religious Reform Movement was reforming the Hindu religion in
North India. It promoted social reform. It made an effort to improve women's conditions. It
promoted social equality while combating untouchability and the rigidities of the hereditary
caste system.
Ramakrishna Mission: It was initiated by Swami Vivekananda in 1897 to spread the
universal message of Vedanta in the light of Sri Ramakrishna Paramahamsa (He was a
teacher of Swami Vivekanand). The main motto of this initiative is to work for the alleviating
of the poor and the downtrodden with the motto ‘for one’s liberation and the good of the
world’.

Therefore, option (b) is the correct answer.

Question 38. + 2.0 - 0.66

Which one of the following was not a provision of the Indian Independence Act of 1947?

A. It abolished the office of Viceroy.

It deprived the Governor General of the power to assent a bill in the name of British
 B.
Crown.

 C. It dissolved the existing Central Legislative Assembly.

D. It continued the services of civil servants appointed by the Secretary of State.

Question Analytics
4385 USERS 946 USERS 3439 USERS 54.4 SECS

ATTEMPTED  SOLVED CORRECTLY  SOLVED INCORRECTLY  YOUR TIME

53.45 SECS 0.0 SECS

 AVG. SOLVING TIME  FASTEST SOLVING TIME

Explanation :

On July 5, 1947, the British Parliament passed the Indian Independence Act which was based
on the Mountbatten Plan (June 3, 1947) and the Act got royal assent on July 18, 1947. The Act
was implemented on August 15, 1947.
Some of the key features of the act are given below:
It ended British rule in India and declared India as an independent and sovereign
state on August 15, 1947.
It provided for the partition of India and the creation of two independent dominions of
India and Pakistan with the right to secede from the British Commonwealth.
Each dominion was to have a Governor-General to be responsible for the effective
operation of the Act. The Constituent Assembly of each new dominion was to exercise
the powers of the legislature of that dominion and the existing Central Legislative
Assembly and the Council of States were to be automatically dissolved.
It abolished the office of Viceroy and provided, for each dominion, a Governor
General, who was to be appointed by the British King on the advice of the dominion
cabinet.
It deprived the British Monarch of his right to veto bills or ask for reservation of certain
bills for his approval. But, this right was reserved for the Governor General. The
Governor General would have full power to assent to any bill in the name of His
Majesty.
It abolished the office of the Secretary of State for India and transferred its functions
to the Secretary of State for Commonwealth Affairs.
It discontinued the appointment to civil services and reservation of posts by the
Secretary of State for India. The members of the civil services appointed before
August 15, 1947, would continue to enjoy all benefits that they were entitled to till
that time.

Therefore, option (b) is the answer.

Question 39. + 2.0 - 0.66

Consider the following statements:

Statement-I:
The modern day on the Earth is longer than a century ago.

Statement-II:

The Earth’s rotation is slowing down due to the tidal effect of the Moon.
Which one of the following is correct in respect of the above statements?

Both Statement-I and Statement-II are correct and Statement-II is the correct
  A.
explanation for Statement-I

Both Statement-I and Statement-II are correct and Statement-II is not the correct
B.
explanation for Statement-I

C. Statement-I is correct but Statement-II is incorrect

D. Statement-I is incorrect but Statement-II is correct

Question Analytics
3721 USERS 1704 USERS 2017 USERS 127.1 SECS

ATTEMPTED  SOLVED CORRECTLY  SOLVED INCORRECTLY  YOUR TIME

42.7 SECS 0.0 SECS

 AVG. SOLVING TIME  FASTEST SOLVING TIME

Explanation :

Earth rotates on its own axis from West to East. As viewed from the North Star or Pole Star
the Earth turns anti-clockwise. The Earth’s axis of rotation meets its surface at the North Pole,
in the northern hemisphere. The South Pole is the other point where the axis of rotation
intersects its surface, in Antarctica. The rotation of the Earth is responsible for the alternate
periods of light and darkness that give us day and night.
The Earth rotates once every twenty-four hours to the Sun and once every 23 hours 56 minutes
and 4 seconds to the stars. Earth’s rotation is slightly slowing with time, thus the day was
shorter in the past, this is due to the tidal effect of the Moon on the Earth’s rotation. The
atomic clock shows that the modern day is longer by about 1.7 milliseconds than a
century ago, slowly increasing the rate at which UTC (Coordinated Universal Time) is
adjusted by leap seconds. So, statements 1 and 2 are correct.

So, both Statement-I and Statement-II are correct and Statement-II is the correct explanation
for Statement-I.
Therefore, option (a) is the correct answer.

Question 40. + 2.0 - 0.66

The term ‘Big Crunch’ is sometimes seen in the news in the context of:

A. study of the Moon’s south pole

B. origin and expansion of the universe from a single point

C. impact of solar flares on satellites revolving around the Earth

  D. collapse of the universe back into a singular point

Question Analytics
3599 USERS 2252 USERS 1347 USERS 56.5 SECS

ATTEMPTED  SOLVED CORRECTLY  SOLVED INCORRECTLY  YOUR TIME


31.23 SECS 0.0 SECS

 AVG. SOLVING TIME  FASTEST SOLVING TIME

Explanation :

The term 'Big Crunch' refers to a scenario in cosmology where the expansion of the
universe eventually slows down and reverses, leading to the collapse of the universe back
into a singular point. This is a contrasting concept to the more commonly discussed idea of
the universe's continuous expansion.
As per the standard expectation, the future expansion of space on cosmic scales is
exponentially rapid. But there is an unlikely, though not inconceivable, possibility that at some
untold date, space’s expansion will reverse and the universe will shrink down to a “Big
Crunch”. In some theories, the Big Crunch precedes another Big Bang and the cycle continues
interminably.

Therefore, option (d) is the correct answer.

Knowledge Box

Theories regarding the universe:

Quantum field theory offers a vision, suggesting that the Higgs field, which permeates the
universe, could undergo a sudden drop in energy configuration due to quantum effects,
leading to the abrupt cessation of the known universe.
There is also the possibility of the decay of protons, which could impact the longevity of
celestial bodies. While current theories suggest a proton's lifetime exceeding 10^73 years,
some models propose decay as early as 10^35 years. In such a scenario, white dwarfs and
neutron stars might persist until 10^39 years before disappearing into decay products.
The fate of the universe is also influenced by the unknown nature of dark matter, adding an
element of uncertainty to its ultimate trajectory.

Relevance: Recently, a physicist traced the universe’s past and future, from a raucous infancy to a
black-hole-dominated aeon of darkness.

Question 41. + 2.0 - 0.66

Which one of the following ecological corridors connects the Eastern and Western Ghats?

A. Javadi Hills
B. Sahyadri Hills

 C. Biligiriranga Hills

 D. Nallamala Hills

Question Analytics
4269 USERS 1659 USERS 2610 USERS 26.6 SECS

ATTEMPTED  SOLVED CORRECTLY  SOLVED INCORRECTLY  YOUR TIME

35.55 SECS 0.0 SECS

 AVG. SOLVING TIME  FASTEST SOLVING TIME

Explanation :

The Eastern Ghats are a discontinuous range of mountains along India's eastern coast. They run
from northern Odisha through Andhra Pradesh to Tamil Nadu in the south passing some parts of
Karnataka. Western Ghats lie parallel to the western coast. They are continuous and can be
crossed through passes only. The Western Ghats are higher than the Eastern Ghats. Their average
elevation is 900– 1600 metres as against 600 metres of the Eastern Ghats.

The Biligiriranga Hills, which run east from the Western Ghats to the River Kaveri, form a
forested ecological corridor that connects the Eastern and Western Ghats. This allows the
second-largest wild Asian elephant population in India to range between the South Eastern
Ghats, the Biligiriranga Hills, the Nilgiri Hills and the South Western Ghats.
The Eastern Ghats are older than the Western Ghats and have a complex geologic history
related to the assembly and breakup of the ancient supercontinent of Rodinia and the assembly
of the Gondwana supercontinent.
The structure of the Eastern Ghats includes thrusts and strike-slip faults all along its range.
They are made up of charnockites, granite gneisses, khondalites, metamorphic gneisses and
quartzite rock formations.

Therefore, option (c) is the correct answer.

Relevance: Recently, a National conference on ‘Tribal Heritage of Eastern Ghats’ was organized in
Visakhapatnam.

Question 42. + 2.0 - 0.66

Consider the following:


1. Sugarcane and Onion

2. Maize and Pigeon pea


3. Cotton and Barley

4. Groundnut and Pigeon pea


How many of the above crops are grown together using intercropping methods in India?

A. Only one

B. Only two

 C. Only three

D. All four

Question Analytics
2898 USERS 822 USERS 2076 USERS 28.2 SECS

ATTEMPTED  SOLVED CORRECTLY  SOLVED INCORRECTLY  YOUR TIME

35.54 SECS 0.0 SECS

 AVG. SOLVING TIME  FASTEST SOLVING TIME

Explanation :

Intercropping: It is a cropping system in which intercrop is grown in between rows of the major
crop, without affecting the optimum plant population as well as the yield of the major crop (base
crop). In this system, two or more crops are grown simultaneously on the same piece of land with
a distinct row arrangement.

Onion crop is best suited for intercrop with paired row planting of sugarcane (November -
December planting) under a drip irrigation system. It saves 25-30% water. So, point 1 is
correct.
Row cropping is practiced for maize and pigeon pea which increases their yield. So, point 2
is correct.
Cotton being a Kharif crop is not intercropped with barley, a rabi crop. So, point 3 is not
correct.
Double-up legume is a method of intercropping two legumes (in this case groundnut and
pigeon pea) that have different growth habits and take advantage of beneficial interactions of
the two legumes on the same piece of land increasing their productivity. Pigeon peas are
suitable for intercropping with groundnut because of their slow growth in the first two
months and only start rapid growth when groundnut approaches maturity. So, point 4 is
correct.
Cultivating black pepper, nutmeg and cinnamon in the coconut garden increases crop
production and provides additional income.

So, only three of the above crops are grown together using intercropping methods in India.

Therefore, option (c) is the correct answer.

Knowledge Box

Benefits of intercropping include:


Better utilisation of growth resources like light, nutrients and moisture
Serves as insurance against failure of any one of the component crops, thus providing
an economy of space and time
Improves soil fertility, reduces soil crust formation and controls soil erosion
Control of pests and diseases and suppression of weeds
Cultivation practices for main crops supplement the requirement of companion crops
giving additional yield from the unit area
Care should be taken to select crops with different growth habits, root growth, duration and
families. The following points are to be considered while selecting crops for intercropping:
Legume crops with non-legume crops
Crops selected should be of different families to avoid pests and diseases
Short-duration crops with long-duration crops
Tall crops with short ones and busy crops with erect ones

Relevance: Recently, Nekram Sharma, a Himachal farmer was awarded Padma Shri for reviving the
traditional ‘nau-anaj’ intercropping method.

Question 43. + 2.0 - 0.66

Consider the following:

1. Subtropical High Pressure Belt


2. Subpolar Low Pressure Belt

3. Equatorial Low Pressure Belt


4. Polar High Pressure Belt

How many of the above are primarily thermally induced pressure belts?
A. Only one

  B. Only two

C. Only three

D. All four

Question Analytics
4207 USERS 3240 USERS 967 USERS 32.0 SECS

ATTEMPTED  SOLVED CORRECTLY  SOLVED INCORRECTLY  YOUR TIME

43.39 SECS 0.0 SECS

 AVG. SOLVING TIME  FASTEST SOLVING TIME

Explanation :

Thermally-induced pressure belts are primarily formed due to incoming solar radiation. These
include the Equatorial Low Pressure Belt and Polar High Pressure Belt. So, points 3 and 4
are correct.
While on the other hand, dynamically-induced pressure belts are primarily formed due to the
rotation of Earth. These include the Subtropical High Pressure Belt and Subpolar Low
Pressure Belt. So, points 1 and 2 are not correct.
Equatorial Low Pressure Belt:
This pressure belt is located in the vicinity of the equator between 10° N and 10° S
latitudes. The average pressure in this belt is less than 1013 millibars throughout.
As these regions receive higher angled rays of the sun throughout the year,
temperatures are high. So, the ground gets heated and the air near the earth’s
surface gets warmed by out radiation. As warm air rises, it comes under lower
pressure and expands. The rising air creates a lower pressure on the Earth’s
surface. Doldrums are associated with this belt.
Polar High Pressure Belts or Polar Highs:
The polar regions in both hemispheres are characterised by extremely low
temperatures throughout the year. Hence high pressure persists at the poles in
these regions. Winds from these belts blow towards sub-polar low-pressure belts
in both hemispheres.
Subtropical High Pressure Belt:
They are located between 25° to 35° latitudes in both hemispheres. The most
important feature of this pressure belt is that it is broken into a number of high-
pressure cells.
Subpolar Low Pressure Belts:
They are located between 60° to 70° latitudes in both hemispheres. These are also
primarily dynamically induced in spite of the fact that these regions experience
low temperatures throughout the year.

So, only two of the above are primarily thermally induced pressure belts.

Therefore, option (b) is the correct answer.

Question 44. + 2.0 - 0.66

Consider the following pairs:

SI. No. Famous Place River

1. Gaya : Phalgu

2. Ujjain : Shipra

3. Nashik : Krishna

4. Madurai : Cauvery

How many pairs given above are correctly matched?

A. Only one pair

 B. Only two pairs

 C. Only three pairs

D. All four pairs

Question Analytics
3391 USERS 1738 USERS 1653 USERS 169.7 SECS

ATTEMPTED  SOLVED CORRECTLY  SOLVED INCORRECTLY  YOUR TIME

34.14 SECS 0.0 SECS

 AVG. SOLVING TIME  FASTEST SOLVING TIME

Explanation :

Gaya in Bihar lies along the Phalgu River, a tributary of the Ganga. It is situated near the
junction of the Gangetic Plain and the Chota Nagpur plateau. The town of Bodh Gaya, south of
Gaya, is famous as the site of the Buddha’s enlightenment. So, pair 1 is correctly matched.
The historical city of Ujjain is located on the river Shipra, which is a tributary of the
Chambal. This city has a Jyotirling, one city out of seven salvation-providing cities,
Gadhkalika and Harsiddhi, the two Shakti Peeths, and sacred Kumbh that takes place in four
cities of India. The cave of King Bhartari is found here. So, pair 2 is correctly matched.
Nashik is an ancient city in the northwest region of Maharashtra. It is situated on the banks of
the Godavari River. Nashik is best known for being one of the Hindu pilgrimage sites, the
Kumbh Mela which is held every 12 years. It is called the ‘Wine Capital of India’ as half of
India’s vineyards and wineries are located in Nashik. So, pair 3 is not correctly matched.
Madurai in Tamil Nadu is located on the Vaigai River. The compact old part of the city—
enclosed by the Anai, Naga and Pasu (Elephant, Snake and Cow) hills—is centred on
Meenakshi Amman (Minakshi-Sundareshwara) Temple. So, pair 4 is not correctly
matched.

So, only two of the above pairs are correctly matched.

Therefore, option (b) is the correct answer.

Question 45. + 2.0 - 0.66

Consider the following:

1. Gulf of Aden

2. Iberian Peninsula
3. Persian Gulf

4. Sinai Peninsula
How many of the above surround the Red Sea?

A. Only one

 B. Only two

 C. Only three

D. All four

Question Analytics
4029 USERS 2671 USERS 1358 USERS 53.4 SECS
ATTEMPTED  SOLVED CORRECTLY  SOLVED INCORRECTLY  YOUR TIME

43.22 SECS 0.0 SECS

 AVG. SOLVING TIME  FASTEST SOLVING TIME

Explanation :

The Red Sea is the world’s northernmost tropical sea and is also one of the most heavily
travelled waterways.
It is connected to the Arabian Sea and the Indian Ocean to the south through the Gulf of Aden
and the narrow strait of Bab-el-Mandeb. The northern portion of the Red Sea is bifurcated
by the Sinai Peninsula into the Gulf of Aqaba and the Gulf of Suez, where it is connected to
the Mediterranean Sea via the Suez Canal. So, points 1 and 4 are correct.
The Persian Gulf does not surround the Red Sea. So, point 3 is not correct.
A total of 6 countries of Asia and Africa border the Red Sea. It is bordered by Yemen and
Saudi Arabia to the east, Egypt to the north and west and Sudan, Eritrea and Djibouti to the
west.
The Iberian Peninsula is located on Europe’s southwestern tip. It is part of the southern
Europe peninsula, which comprises three peninsulas; Iberian, Balkan, and Italian peninsulas.
The Iberian is the westernmost peninsula of the three peninsulas. Its southern tip is separated
from the African continent by the narrow Strait of Gibraltar. At the narrowest point, the
peninsula is only 8 kilometers from Africa, specifically Morocco. So, point 2 is not correct.

So, only two of the above surround the Red Sea.


Therefore, option (b) is the correct answer.

Relevance: Militant attacks in the Red Sea and surging freight rates are causing a split in the global
oil market.

Question 46. + 2.0 - 0.66

Which of the following countries’ spacecraft have made a landing on the Moon?
1. Japan

2. United Kingdom
3. China

4. France

5. South Korea
Select the correct answer using the code given below:

  A. 1 and 3 only

B. 1, 2 and 5 only

C. 2, 3 and 4 only

D. 2, 3, 4 and 5 only

Question Analytics
4597 USERS 3739 USERS 858 USERS 112.2 SECS

ATTEMPTED  SOLVED CORRECTLY  SOLVED INCORRECTLY  YOUR TIME

42.04 SECS 0.0 SECS

 AVG. SOLVING TIME  FASTEST SOLVING TIME

Explanation :

Only a few countries- the United States of America (USA), Russia (formerly the USSR),
China, India, and Japan have successfully landed on the Moon using an impactor or a lander.
A lander is a spacecraft designed to land on a celestial body while an impactor gets destroyed
after landing.
In 2024, Japan's Smart Lander for Investigating Moon (SLIM) became the fifth country to
soft-land on the Moon. So, point 1 is correct.
China has successfully landed on the Moon twice, in 2013 and 2019. Chang'e 4 was launched
in 2018 and landed on the moon in 2019. It was the first probe to land on the Moon's far side.
So, point 3 is correct.
The United Kingdom, France and South Korea have not sent a spacecraft to the Moon. So,
points 2, 4 and 5 are not correct.
Recently, India made history by becoming the first country to touch down near the little-
explored South Pole region and join the USA, Russia (Soviet Union) and China in achieving
a moon landing.

Therefore, option (a) is the correct answer.

Question 47. + 2.0 - 0.66

Consider the following statements:


1. Quantum dots are fluorescent nanocrystals with a semiconductor material at its core.

2. Smaller quantum dots emit high-frequency photons with shorter wavelengths.


Which of the statements given above is/are correct?

A. 1 only

B. 2 only

  C. Both 1 and 2

D. Neither 1 nor 2

Question Analytics
3623 USERS 2860 USERS 763 USERS 79.1 SECS

ATTEMPTED  SOLVED CORRECTLY  SOLVED INCORRECTLY  YOUR TIME

30.2 SECS 0.0 SECS

 AVG. SOLVING TIME  FASTEST SOLVING TIME

Explanation :

The Nobel Prize in Chemistry 2023 rewards the discovery and development of Quantum Dots
(QDs), nanoparticles so tiny that their size determines their properties.
QDs are fluorescent semiconductor nanocrystals, composed of materials from the elements
in the periodic groups of II–VI, III–V or IV–VI, e.g. admium telluride (Cd from group II and
Te from group VI) and indium phosphamide (In from group III and P from group V). They
range in size from 2 to 10 nm in diameter and contain approximately 200–10,000 atoms. So,
statement 1 is correct.
QDs emit light and the size of nanocrystal determines the wavelength of light emitted by it. A
smaller quantum dot emits high-frequency photons with shorter wavelengths. So,
statement 2 is correct.
When particles are just a few nanometres in diameter, the space available to electrons
shrinks. This affects the particle’s optical properties. The luminous properties of
quantum dots are utilised in computer and television screens based on QLED
technology, where the Q stands for quantum dot.

Therefore, option (c) is the correct answer.

Relevance: The Nobel Prize in Chemistry 2023 was awarded to Moungi G. Bawendi, Louis E. Brus
and Aleksey Yekimov "for the discovery and synthesis of quantum dots".
Question 48. + 2.0 - 0.66

Consider the following:

1. Telecom service providers


2. Internet service providers

3. Search engines

4. Online payment sites


5. Cyber cafes

How many of the above are considered as ‘Intermediaries’ under the Information Technology Act,
2000?

A. Only two

B. Only three

 C. Only four

 D. All five

Question Analytics
3888 USERS 1121 USERS 2767 USERS 78.4 SECS

ATTEMPTED  SOLVED CORRECTLY  SOLVED INCORRECTLY  YOUR TIME

39.03 SECS 0.0 SECS

 AVG. SOLVING TIME  FASTEST SOLVING TIME

Explanation :

Information Technology Act, 2000 is the primary law in India dealing with cybercrime and
electronic commerce. It defines an intermediary as a person who receives, stores or transmits
any electronic record and provides any service relating to such record on the behalf of another
person.
Intermediaries include network service providers, telecom service providers, internet
service providers, search engines, web-hosting service providers, online-auction sites,
online payment sites, online-marketplaces and cyber cafes. So, points 1, 2, 3, 4 and 5
are correct.
Intermediaries perform functions such as hosting content, collecting information and
evaluating information, facilitating communication and information exchange, facilitating the
use of the internet etc. Examples of intermediaries include social media platforms such as
WhatsApp, Twitter, Instagram and Facebook; e-commerce sites such as Myntra and Amazon;
search engines, cloud service providers etc.

So, all five of the above are considered as ‘Intermediaries’ under the Information Technology
Act, 2000.

Therefore, option (d) is the correct answer.

Question 49. + 2.0 - 0.66

Consider the following statements:


1. Artificial General Intelligence (AGI) is a type of artificial intelligence that is limited to a specific
or narrow area.

2. Natural Language Processing (NLP) enables computers to analyse and understand human
language, both written and spoken.

Which of the statements given above is/are correct?

A. 1 only

  B. 2 only

C. Both 1 and 2

D. Neither 1 nor 2

Question Analytics
3962 USERS 2668 USERS 1294 USERS 22.3 SECS

ATTEMPTED  SOLVED CORRECTLY  SOLVED INCORRECTLY  YOUR TIME

32.89 SECS 0.0 SECS

 AVG. SOLVING TIME  FASTEST SOLVING TIME

Explanation :

Artificial General Intelligence (AGI), refers to a proposed type of artificial intelligence that
has the ability to understand, learn and apply its intelligence to a wide range of problems,
much like a human being.
Weak Artificial Intelligence (AI) — also called narrow AI — is a type of artificial
intelligence that is limited to a specific or narrow area.
Unlike narrow or weak AI, which is designed to perform specific tasks (like image recognition,
language translation, or playing chess), AGI can theoretically perform any intellectual task
that a human being can and more. Statement 1 is not correct.
Natural Language Processing (NLP) is a field of Artificial Intelligence (AI) that enables
computers to analyze and understand human language, both written and spoken. It was
formulated to build software that generates and comprehends natural languages so that a user
can have natural conversations with a computer instead of through programming or artificial
languages like Java or C. So, statement 2 is correct.
NLP combines computational linguistics—rule-based modeling of human language—
with statistical and machine learning models to enable computers and digital devices to
recognize, understand and generate text and speech.

Therefore, option (b) is the correct answer.


Relevance: The Competition Commission of India will soon initiate a market study on Artificial
Intelligence (AI) to develop a comprehensive understanding of its evolving landscape and use cases
in markets in India.

Question 50. + 2.0 - 0.66

Consider the following:

1. Maglev trains

2. Experimental nuclear fusion reactors


3. Cell phone base stations

4. Particle detectors
Applications of superconductivity can be used in how many of the above fields?

A. Only one

 B. Only two

C. Only three

 D. All four

Question Analytics
3601 USERS 2694 USERS 907 USERS 25.0 SECS
ATTEMPTED  SOLVED CORRECTLY  SOLVED INCORRECTLY  YOUR TIME

27.05 SECS 0.0 SECS

 AVG. SOLVING TIME  FASTEST SOLVING TIME

Explanation :

A superconductor is a material that achieves superconductivity, which is a state of matter that has no
electrical resistance and does not allow magnetic fields to penetrate. An electric current in a
superconductor can persist indefinitely. Superconductivity can only typically be achieved at very cold
temperatures. The credit for the discovery of superconductivity goes to Dutch physicist Heike
Kamerlingh Onnes.

In order to generate the strong magnetic fields used in Magnetic Resonance Imaging (MRI)
and Nuclear Magnetic Resonance Imaging (NMRI), the machines use powerful
electromagnets.
Similar superconducting electromagnets are also used in maglev trains, experimental nuclear
fusion reactors and high-energy particle accelerator laboratories. Superconductors are also
used to power railguns and coilguns, cell phone base stations, fast digital circuits and
particle detectors. So, points 1, 2, 3 and 4 are correct.

So, all four of the above are the applications of superconductivity.

Therefore, option (d) is the correct answer.

Relevance: The scientific community now firmly believes that LK-99 is not a superconductor
operating at room temperature.

Question 51. + 2.0 - 0.66

Consider the following:

1. NITI Aayog

2. Cabinet Secretariat
3. National Disaster Management Authority

4. Central Bureau of Investigation


How many of the above are neither Constitutional nor statutory bodies/institutions in India?

A. Only one

 B. Only two
 C. Only three

D. All four

Question Analytics
4704 USERS 1461 USERS 3243 USERS 104.2 SECS

ATTEMPTED  SOLVED CORRECTLY  SOLVED INCORRECTLY  YOUR TIME

43.25 SECS 0.0 SECS

 AVG. SOLVING TIME  FASTEST SOLVING TIME

Explanation :

NITI Aayog: It was formed via a resolution of the Union Cabinet on 1 January 2015. It is
chaired by the Prime Minister. The Governing Council of NITI Aayog, comprising Chief
Ministers of all the States and Union Territories with legislatures and Lt Governors of other
Union Territories, came into effect on 16 February 2015 via a notification by the Cabinet
Secretariat. So, point 1 is correct.
Cabinet Secretariat: It is responsible for the administration of the Government of India
(Transaction of Business) Rules, 1961 and the Government of India (Allocation of
Business) Rules 1961, facilitating smooth transaction of business in Ministries/ Departments
of the Government. This Secretariat provides Secretarial assistance to the Cabinet and its
Committees, and also assists in decision-making in Government. It is neither a Constitutional
nor statutory institution. So, point 2 is correct.
National Disaster Management Authority (NDMA): It is established under the Disaster
Management Act, 2005 to spearhead and implement a holistic and integrated approach to
disaster management in the country. Initially, the NDMA was constituted in 2005 by an
Executive Order of the Government of India. Subsequently, the NDMA was notified in 2006
under the provisions of the Act. So, point 3 is not correct.
Central Bureau of Investigation: It was set up in 1963 by a resolution of the Ministry of
Home Affairs. Later, it was transferred to the Ministry of Personnel, Public Grievances and
Pensions and now it enjoys the status of an attached office. The Special Police Establishment
(which looked into vigilance cases) setup in 1941 was also merged with the CBI. So, point 4 is
correct.

So, only three of the above are neither Constitutional nor statutory bodies/institutions in India.

Therefore, option (c) is the correct answer.


Question 52. + 2.0 - 0.66

Consider the following statements with reference to the Joint Sitting of the Parliament:

1. It is obligatory for the President to summon the Houses to meet in a joint sitting when there is a
deadlock over the passage of a Bill.

2. Once the President has notified his intention to summon the Houses for a joint sitting, dissolution
of Lok Sabha does not affect the proceeding with the Bill.
Which of the statements given above is/are correct?

A. 1 only

 B. 2 only

 C. Both 1 and 2

D. Neither 1 nor 2

Question Analytics
4559 USERS 1401 USERS 3158 USERS 43.3 SECS

ATTEMPTED  SOLVED CORRECTLY  SOLVED INCORRECTLY  YOUR TIME

48.24 SECS 0.0 SECS

 AVG. SOLVING TIME  FASTEST SOLVING TIME

Explanation :

Joint sitting of the Parliament is an extraordinary machinery provided by the Constitution of India to
resolve a deadlock between the two Houses over the passage of a Bill.

This provision is only an enabling one, empowering the President to take a step for resolving a
deadlock between the two Houses. It is not obligatory upon him to summon the Houses to
meet in a joint sitting. Moreover, this provision does not disable the receiving House from
passing the Bill after the lapse of six months, provided the Bill has not lapsed by reason of
dissolution or the President has not already notified his intention to convene a joint sitting. So,
statement 1 is not correct.
A Bill upon which the Houses have disagreed and the President has notified his intention to
summon a Joint Sitting of the Houses to consider the Bill prior to dissolution does not lapse
on the dissolution of the Lok Sabha.
Once the President has notified his intention to summon the Houses for a joint sitting,
dissolution of Lok Sabha does not stand in the way of proceeding with the Bill at a joint
sitting. So, statement 2 is correct.
The Secretary General, Lok Sabha, issues summons to each member of Lok Sabha and Rajya
Sabha, specifying the time and place fixed by the President for the joint sitting. At a joint
sitting, the Speaker of Lok Sabha presides and the Secretary-General, Lok Sabha acts as
Secretary-General of the joint sitting.

Therefore, option (b) is the correct answer.

Relevance: President of India addressed a joint sitting of Lok Sabha and Rajya Sabha to mark the
start of the Budget Session of Parliament.

Question 53. + 2.0 - 0.66

Consider the following statements with reference to the Indian Judiciary:

1. A Supreme Court judge can be removed only on the ground of violation of the Constitution.
2. A retired Supreme Court Judge cannot take up post-retirement appointments from the Government
of India.

3. A retired High Court Judge cannot plead in any High Court within the territory of India.

How many of the above statements are correct?

 A. Only one

B. Only two

C. All three

 D. None

Question Analytics
4615 USERS 1304 USERS 3311 USERS 96.4 SECS

ATTEMPTED  SOLVED CORRECTLY  SOLVED INCORRECTLY  YOUR TIME

56.94 SECS 0.0 SECS

 AVG. SOLVING TIME  FASTEST SOLVING TIME

Explanation :

A Judge of the Supreme Court can be removed from his Office by an order of the President.
The President can issue the removal order only after an address by Parliament has been
presented to him in the same session for such removal. The address must be supported by a
special majority of each House of Parliament. The grounds of removal are two: proved
misbehaviour or incapacity. So, statement 1 is not correct.
Article 124 (7) of the Constitution of India provides that the retired judges of the Supreme
Court are prohibited from pleading or acting in any Court or before any authority within the
territory of India. However, at present, there is no law that prevents a retired judge of the
Supreme Court or High Courts from taking up a post-retirement appointment from the
Government of India. So, statement 2 is not correct.
The retired permanent judges of a High Court are prohibited from pleading or acting in any
court or before any authority in India except the Supreme Court and the other High Courts.
So, statement 3 is not correct.

So, none of the above statements are correct.

Therefore, option (d) is the correct answer.

Relevance: Recently, the Supreme Court of India dismissed a public interest litigation (PIL) petition
seeking a ‘cooling off’ period of two years before any retired judge of a constitutional court can
accept a post-retirement appointment.

Question 54. + 2.0 - 0.66

Which of the following form the Selection Committee to appoint members of the Election
Commission of India?

 A. Prime Minister, Leader of Opposition in Lok Sabha and Chief Justice of India

B. Prime Minister, Chief Justice of India and Union Cabinet Minister

 C. Prime Minister, Leader of Opposition in Lok Sabha and Union Cabinet Minister

Prime Minister, Leader of Opposition in Lok Sabha and Leader of Opposition in Rajya
D.
Sabha

Question Analytics
4853 USERS 3748 USERS 1105 USERS 69.1 SECS

ATTEMPTED  SOLVED CORRECTLY  SOLVED INCORRECTLY  YOUR TIME

30.15 SECS 0.0 SECS

 AVG. SOLVING TIME  FASTEST SOLVING TIME

Explanation :
The Chief Election Commissioner and Other Election Commissioners (Appointment,
Conditions of Service and Term of Office) Act, 2023 governs the procedure for appointment
of Chief Election Commissioner and Election Commissioners of the Election Commission of
India.
The Chief Election Commissioner and other Election Commissioners shall be appointed
by the President by warrant under his hand and seal. A Search Committee headed by the
Minister of Law and Justice and comprising two other members not below the rank of
Secretary to the Government of India shall prepare a panel of five persons for consideration of
the Selection Committee.
The Chief Election Commissioner and other Election Commissioners shall be appointed by the
President on the recommendation of a Selection Committee consisting of:
Prime Minister—Chairperson
Leader of Opposition in the Lok Sabha or Member, where the Leader of Opposition in
the Lok Sabha has not been recognised as such, the leader of the single largest party in
opposition of the Government in the House of the People shall be deemed to be the
Leader of Opposition
Union Cabinet Minister to be nominated by the Prime Minister.
Before this Act was passed, the Supreme Court in 2023 ruled that the appointment of the
Chief Election Commissioner and the Election Commissioners shall be made by the President
on the advice of a Committee consisting of the Prime Minister, the Leader of the Opposition
of the Lok Sabha, and in case no Leader of the Opposition is available, the leader of the
largest opposition Party in the Lok Sabha in terms of numerical strength, and the Chief Justice
of India. However, the Court was careful to specify that these norms were “subject to any law
to be made by Parliament’. Parliament accordingly passed the Chief Election Commissioner
and Other Election Commissioners (Appointment, Conditions of Service and Term of Office)
Act, 2023.

Therefore, option (c) is the correct answer.


Relevance: Recently, Parliament has passed the 'Chief Election Commissioner and other Election
Commissioners (Appointment, Conditions of Service and Term of Office) Bill, 2023'.

Question 55. + 2.0 - 0.66

Consider the following statements:


1. A summon is issued to call a person to appear before the court whereas a warrant empowers law
enforcement officials to take specific actions.

2. A summon can be issued by an administrative authority whereas a warrant is a legal document


issued by a judicial officer.

Which of the statements given above is/are correct?

A. 1 only

B. 2 only

  C. Both 1 and 2

D. Neither 1 nor 2

Question Analytics
4516 USERS 3293 USERS 1223 USERS 22.8 SECS

ATTEMPTED  SOLVED CORRECTLY  SOLVED INCORRECTLY  YOUR TIME

39.97 SECS 0.0 SECS

 AVG. SOLVING TIME  FASTEST SOLVING TIME

Explanation :

A summon is a formal legal notice issued by a court or a judicial officer to call a person to
appear before the court in both criminal and civil cases. It serves as a means to notify the
accused (in criminal cases) or the defendant (in civil cases) about the charges or claims brought
against them. On the other hand, a warrant is a formal legal document issued by a court or a
judicial authority that empowers law enforcement officials to take specific actions. So,
statement 1 is correct.
In the context of criminal cases, a warrant can be an “arrest warrant,” which authorises
the police to apprehend a specific individual accused of committing a crime.
Alternatively, it can be a “search warrant,” granting the authorities permission to conduct
a search at a specified location to gather evidence related to a crime.
A summons is also a legal document that notifies an individual or entity that they are required
to appear in court or respond to a legal proceeding. It is issued by a court or administrative
agency and typically informs the recipient of the date, time, and location they need to appear.
A warrant is a document issued by a Court or judicial officer to a person or an entity
involved in a legal proceeding. A warrant is issued only in serious offences and/or after
duly served summons is disobeyed or if the accused has wilfully avoided the services of
the summons. So, statement 2 is correct.

Therefore, option (c) is the correct answer.

Question 56. + 2.0 - 0.66

Consider the following:

1. Holders of Bharat Ratna decoration

2. Cabinet Secretary
3. Judges of High Courts

4. Deputy Chief Ministers of States


5. Ambassadors of foreign countries

How many of the above are exempted from security checks at the airports in India?

A. Only two

B. Only three

  C. Only four

D. All five

Question Analytics
3251 USERS 652 USERS 2599 USERS 52.2 SECS

ATTEMPTED  SOLVED CORRECTLY  SOLVED INCORRECTLY  YOUR TIME

45.3 SECS 0.0 SECS

 AVG. SOLVING TIME  FASTEST SOLVING TIME

Explanation :

The Ministry of Civil Aviation gives exemption from the pre-embarkation security checks in
consultation with or after receiving inputs from the Ministry of Home Affairs (MHA). The list
of VVIPs/VIPs exempt from pre-embarkation security check is as follows:
President, Vice President, Prime Minister, Governors of States, Former Presidents,
Former Vice-President, Chief Justice of India, Speaker of Lok Sabha, Union Ministers of
Cabinet rank, Chief Ministers of States, Deputy Chief Ministers of States, NITI Aayog,
Leader of Opposition in Lok Sabha & Rajya Sabha, Holders of Bharat Ratna
Decoration, Ambassadors of foreign countries, Charge D` Affairs and High
Commissioners and their spouses, Judges of Supreme Court, Chief Election
Commissioner, Comptroller & Auditor General of India, Deputy Chairman of Rajya
Sabha & Deputy Speaker of Lok Sabha, Minister of State of the Union Council of
Ministers, Attorney General of India, Cabinet Secretary, Lt. Governors of Union
Territories, Chiefs of staffs holding the rank of full General or equivalent rank, Chief
Justices of the High Courts, Chief Ministers of Union Territories, Deputy Chief
Ministers of Union Territories, Visiting Foreign dignitaries of the above status, H.H. the
Dalai Lama. So, points 1, 2, 4 and 5 are correct.
Only Chief Justices of High Courts are exempted from security checks and not other
Judges of High Courts. So, point 3 is not correct.

So, only four of the above are exempted from security checks at the airports in India.

Therefore, option (c) is the correct answer.

Question 57. + 2.0 - 0.66

Consider the following statements with reference to the Governor of a State in India:
1. The Draft Constitution of India provided for the direct election of the Governor based on universal
adult suffrage.
2. The Constitution of India requires the President to consult the Chief Minister of the concerned
State while appointing the Governor.

Which of the statements given above is/are correct?

 A. 1 only

B. 2 only

C. Both 1 and 2

D. Neither 1 nor 2

Question Analytics
4577 USERS 1166 USERS 3411 USERS 13.3 SECS

ATTEMPTED  SOLVED CORRECTLY  SOLVED INCORRECTLY  YOUR TIME

43.06 SECS 0.0 SECS

 AVG. SOLVING TIME  FASTEST SOLVING TIME


Explanation :

The Draft Constitution of India provided for the direct election of the Governor based on
universal adult suffrage. But the Constituent Assembly opted for the present system of
appointment of Governor by the President because the direct election of the Governor is
incompatible with the Parliamentary System established in the States. So, statement 1 is
correct.
Two conventions (not mentioned in the Constitution of India) have also developed with
regards to the appointment of the Governor of a State. First, s/he should be an outsider, that is,
s/he should not belong to the State where s/he is appointed, so that s/he is free from the local
politics. Second, while appointing the Governor, the President is required to consult the
Chief Minister of the State concerned, so that the smooth functioning of the constitutional
machinery in the State is ensured. So, statement 2 is not correct.

Therefore, option (a) is the correct answer.

Relevance: Recently the Chief Justice of India made a stinging rebuke against the governors of
Punjab and Tamil Nadu in response to serious allegations from their respective state governments.

Question 58. + 2.0 - 0.66

Two parties from different States which have the same party symbol are contesting for a State
Legislative Assembly seat. Which one of the two will be granted the party symbol in such a
situation?

A. The party with the longest history of functioning and formation

B. The party with higher vote percentage in their last State Legislative Assembly elections

C. The party with a higher number of Members of Parliament

Neither of the parties will be able to use their party symbols and will be allotted 'free
  D.
symbol'

Question Analytics
4184 USERS 2178 USERS 2006 USERS 66.5 SECS

ATTEMPTED  SOLVED CORRECTLY  SOLVED INCORRECTLY  YOUR TIME

56.4 SECS 0.0 SECS

 AVG. SOLVING TIME  FASTEST SOLVING TIME


Explanation :

In India, in the case where two parties from different States have the same election symbol, the
parties can contest on their election symbols as long as they are not fielded against each
other in a State Legislative Assembly seat. If they are fighting against each other, then they
will not be able to use their party symbols and will be allotted 'free symbols' in those
constituencies.
The candidates of these parties in such constituencies shall be allotted 'free symbol'
from the list of free symbols as per provisions of Symbols (Reservation and Allotment)
Order, 1968.
While National Parties are free to use their 'reserved symbol' across India, the recognised
State Parties can use their symbols in their States. To use the symbol outside their State, they
have to seek permission from the Election Commission of India.

Therefore, option (d) is the correct answer.

Relevance: A political party filed a writ petition challenging the Election Commission of India’s
decision to allot the symbols in a State recently.

Question 59. + 2.0 - 0.66

The Speaker of the State Legislative Assembly acts as a quasi-judicial authority in which one of the
following situations?

A. Certifying if a Bill is Money Bill or not

B. Administering oath of office to newly elected members

C. Facilitating discussions on Legislative Bills proposed by the ruling party

Resolving disputes related to disqualification of members under the Tenth Schedule of


  D.
the Constitution of India

Question Analytics
4619 USERS 4099 USERS 520 USERS 35.2 SECS

ATTEMPTED  SOLVED CORRECTLY  SOLVED INCORRECTLY  YOUR TIME

34.56 SECS 0.0 SECS

 AVG. SOLVING TIME  FASTEST SOLVING TIME

Explanation :
Based on the Constitutional provisions, Rules of Business of the House, and the Conventions,
the Speaker’s powers and functions can be divided into four broad categories which can be to
(a) run the business of the House (b) administrative action (c) quasi-judicial and (d) other
functions.
In the quasi-judicial role, the Speaker decides, under the Tenth Schedule of the Constitution,
on the issue of defection of the members from the political party which influences the
composition of the House and the formation/continuation of the government. While deciding
the defection matters, the principles of natural justice will guide the exercise of power on the
procedural aspect. On the substantive aspect, the Speaker should be guided by the absence of
arbitrariness and the inherent characteristics of impartiality.

Therefore, option (d) is the correct answer.

Knowledge Box

Powers and Functions of Speaker:

The Speaker facilitates the business of the House, ensures equitable participation of every
stakeholder during the discussion, decides on the motions moved by the members, assists the
members to hold the executive accountable, plays the role of a disciplinarian by
suspending/terminating the member or ask them to withdraw from the House, adjourns the
House, expunges the unparliamentary statements and decides on the nature of the Bill.
On the administrative side, the Speaker heads the Lok Sabha Secretariat, exercises power
over a number of Parliamentary Committees such as the Rules Committee, the Business
Advisory Committee and the General Purposes Committee and nominates the chairman of
various committees in place.
In addition to this, few other powers are vested in the Speaker which includes the power to
exercise a casting vote, to resolve a deadlock over a particular matter. That is, when the House
initiates a voting procedure, he does not cast a vote in the first instance but shall have and
exercise a casting vote in the case of an equality of votes.

Question 60. + 2.0 - 0.66

In India, Aadhar is mandatory for which of the following purposes?


1. Payment of wages under MGNREGA scheme

2. To avail benefits under Integrated Child Protection and Maternity Benefits schemes
3. Opening of bank accounts by Non-Resident Indians

4. Filing of Income Tax Return


Select the correct answer using the code given below:

A. 1 and 4 only

B. 2 and 3 only

  C. 1, 2 and 4 only

D. 2, 3 and 4 only

Question Analytics
4529 USERS 2733 USERS 1796 USERS 62.0 SECS

ATTEMPTED  SOLVED CORRECTLY  SOLVED INCORRECTLY  YOUR TIME

46.86 SECS 0.0 SECS

 AVG. SOLVING TIME  FASTEST SOLVING TIME

Explanation :

Aadhaar is a 12-digit unique identification number issued by the Unique Identification


Authority of India (UIDAI) to every individual resident of India. It serves as proof of
identity, which is linked to the individual's biometric and demographic information.
The Central Government has mandated the payment of all MGNREGA (Mahatma Gandhi
National Rural Employment Guarantee Act 2005) wages through an Aadhaar-Based
Payment System (ABPS). All workers are required to furnish their Aadhaar details. So, point
1 is correct.
All Integrated Child Protection and Maternity Benefits schemes mandatorily require the
Aadhaar card. Women who wish to take advantage of loans, projects and training opportunities
provided by the government now need an Aadhaar. So, point 2 is correct.
Aadhaar is not a proof of Indian citizenship and does not grant any foreigner the right to stay in
the country in case they are non-compliant with other requirements. NRIs are not compelled
by any mandate to link their bank accounts with Aadhaar if they do not wish to benefit
under section 7 of the Aadhaar Act. So, point 3 is not correct.
It is mandatory for filing Income Tax Returns (ITR) and allotment of Permanent Account
Number (PAN). Section 139AA of the Income-tax Act, 1961 as introduced by the Finance
Act, 2017 provides for mandatory quoting of Aadhaar/Enrolment ID of Aadhaar application
form, for filing of return of income. So, point 4 is correct.

Therefore, option (c) is the correct answer.

Relevance: The Central Government has mandated the payment of all wages under the Mahatma
Gandhi National Rural Employment Guarantee Act (MGNREGA) scheme through an Aadhaar-
Based Payment System (ABPS).

Question 61. + 2.0 - 0.66

In the context of the United Nations (UN), consider the following statements:

1. The UN Security Council (UNSC) resolutions are legally binding unlike the UN General
Assembly’s.

2. A UNSC resolution must have the affirmative vote of its nine members including the five
permanent members.
3. A UNSC resolution fails if any one of its five permanent members abstains from voting.

Which of the statements given above are correct?

 A. 1 and 2 only

B. 2 and 3 only

C. 1 and 3 only

 D. 1, 2 and 3

Question Analytics
4139 USERS 1436 USERS 2703 USERS 90.6 SECS

ATTEMPTED  SOLVED CORRECTLY  SOLVED INCORRECTLY  YOUR TIME

52.25 SECS 0.0 SECS

 AVG. SOLVING TIME  FASTEST SOLVING TIME

Explanation :

All member countries of the United Nations (UN) are part of its General Assembly. UN
Security Council (UNSC) consists of the United States, the United Kingdom, France, Russia
and China. These are the ‘permanent five’ or P5 countries, called so because the UNSC also
has 10 non permanent members who are elected for two years each based on UN General
Assembly (UNGA) elections. This body can pass resolutions on relevant matters. Only a
simple majority (of more than half the members) is required to pass its resolutions. Also,
UNSC resolutions are legally binding, unlike the UNGA’s. So, statement 1 is correct.
In general, to be adopted, a draft resolution on a non-procedural matter must have the
affirmative vote of nine members of the Council, including the concurring votes of the five
permanent members: China, France, Russian Federation, United Kingdom of Great Britain and
Northern Ireland and the United States of America. So, statement 2 is correct.
A draft does not pass:
If the it fails to win nine votes, or
If a permanent member casts a negative vote (veto)
Each of the P5 members has the power to veto a vote. It was agreed by the drafters of the UN
Charter (which lays down its governing principles) that if any one of the five permanent
members cast a negative vote in the 15-member UNSC, the resolution would fail. A member
can choose to abstain though, allowing the resolution to be adopted if it obtains the
minimum nine votes. So, statement 3 is not correct.

Therefore, option (a) is the correct answer.

Relevance: Recently, the United Nations General Assembly (UNGA) adopted a resolution for an
immediate ceasefire in the Israel-Hamas conflict in Gaza.

Question 62. + 2.0 - 0.66

Consider the following pairs:

Sl. No. Area of conflict mentioned in news Country where it is located

1. Galmudug : Afghanistan

2. Abyei : South Sudan

3. Amhara : Ethiopia

How many of the above pairs are correctly matched?

A. Only one pair

 B. Only two pairs

C. All three pairs

D. None

Question Analytics
1697 USERS 1014 USERS 683 USERS 8.2 SECS
1697 USERS 1014 USERS 683 USERS 8.2 SECS

ATTEMPTED  SOLVED CORRECTLY  SOLVED INCORRECTLY  YOUR TIME

19.14 SECS 0.0 SECS

 AVG. SOLVING TIME  FASTEST SOLVING TIME

Explanation :

Galmudug: It is located in central Somalia and has faced inter-clan conflicts over
resources, land and elections. Recently, the Somali National Army and local soldiers have
been fighting to liberate the country from militant groups like al-Shabaab. So, pair 1 is not
correctly matched.
Abyei: It is a region located between Sudan and South Sudan. Both Sudan and South Sudan
claim ownership of Abyei, whose status was unresolved after South Sudan became
independent from Sudan in 2011. Currently, the region is under the control of South Sudan.
Inter-communal and cross-border clashes have escalated since South Sudan deployed its
troops to Abyei in March 2023. So, pair 2 is correctly matched.
Amhara: It is a region in Ethiopia. Fighting erupted here recently between federal forces and
a local militia called Fano, which has accused the government of undermining the region’s
security. The government declared a state of emergency to respond to the insurgency in the
region. So, pair 3 is correctly matched.

So, only two of the above pairs are correctly matched.

Therefore, option (b) is the correct answer.

Relevance: Recently, Galmudug, Abyei and Amhara have been frequently seen in the news due to
the emergence of local conflicts.

Question 63. + 2.0 - 0.66

Intergovernmental Negotiations (IGN) Framework is often seen in the news in the context of which
one of the following organisations?

  A. United Nations (UN)

B. World Bank (WB)

C. World Economic Forum (WEF)

D. International Monetary Fund (IMF)


Question Analytics
3567 USERS 1799 USERS 1768 USERS 39.5 SECS

ATTEMPTED  SOLVED CORRECTLY  SOLVED INCORRECTLY  YOUR TIME

32.26 SECS 0.0 SECS

 AVG. SOLVING TIME  FASTEST SOLVING TIME

Explanation :

Intergovernmental Negotiations (IGN) Framework is a group of nation-states working


within the United Nations to further reform the United Nations Security Council (UNSC).
Composition: The IGN is composed of several different international organisations, namely:
African Union;
G4 nations (India, along with Brazil, Japan and Germany are pressing for a permanent
seat in the reformed UNSC);
Uniting for Consensus Group (UfC), also known as the "Coffee Club";
L.69 Group of Developing Countries;
Arab League; and
Caribbean Community (CARICOM).
The group's conversations are considered "informal" in nature due to the lack of a single text
and thus, United Nations General Assembly rules of procedure don't apply.

Therefore, option (a) is the correct answer.

Relevance: Recently, France and India called for the early commencement of text-based negotiations
at the Intergovernmental Negotiations (IGN) in the UN.

Question 64. + 2.0 - 0.66

Consider the following statements with reference to Taiwan:


1. It is a representative democracy with a semi-presidential system.

2. It is the largest manufacturer of semiconductors across the World.

3. It is a part of the Chip4 Alliance alongside Japan, South Korea and the United States.
4. India was one of the first countries to establish diplomatic relations with it after World War II.

How many of the above statements are correct?

A. Only one
B. Only two

 C. Only three

D. All four

Question Analytics
3374 USERS 1069 USERS 2305 USERS 5.6 SECS

ATTEMPTED  SOLVED CORRECTLY  SOLVED INCORRECTLY  YOUR TIME

43.18 SECS 0.0 SECS

 AVG. SOLVING TIME  FASTEST SOLVING TIME

Explanation :

Taiwan:

It was established by a Chinese leader named Chiang Kai-shek. After getting defeated by the
Chinese Communist Party, he fled to Taiwan in 1949. It is generally considered to be part of
the People’s Republic of China.
Taiwan has been considered as a semi-presidential country. According to its constitutional
amendments, its President is directly elected, and the Premier (Prime Minister) and Cabinet are
responsible to the Legislature. It has a dual executive system. So, statement 1 is correct.
It manufactures close to sixty percent of the world's semiconductors and over 90 percent of
the most advanced chips that are required for almost all electronic equipment. So, statement 2
is correct.
The “Chip 4” or “Fab 4” alliance includes four of the world’s top producers of
semiconductors namely the United States of America, Japan, Taiwan and South Korea. It
represents more than 70 per cent of the value of the global semiconductor industry. It is aimed
at enhancing the "security" and "resilience" of semiconductor supply chains, including by
reducing the world's reliance on chips made in China. So, statement 3 is correct.
It is not a member of the United Nations. India does not have formal diplomatic ties with
Taiwan but both sides have trade and people-to-people relations. India maintains a One-China
policy. So, statement 4 is not correct.

So, only three of the above statements are correct.


Therefore, option (c) is the correct answer.

Relevance: Recently, India and Taiwan reportedly signed an agreement on Labour cooperation.

Question 65. + 2.0 - 0.66

In which one of the following contexts the terms “Baa2 and Baa3” are mentioned?

A. Quantum computing

  B. Credit ratings

C. Cyber security

D. Gene editing

Question Analytics
4139 USERS 2807 USERS 1332 USERS 22.6 SECS

ATTEMPTED  SOLVED CORRECTLY  SOLVED INCORRECTLY  YOUR TIME

20.54 SECS 0.0 SECS

 AVG. SOLVING TIME  FASTEST SOLVING TIME

Explanation :

A credit rating is an independent assessment of the creditworthiness of a business or


government entity in general terms or with respect to a specific financial obligation, such as a
new bond issue. Baa2 and Baa3 are credit ratings provided by credit rating agencies.
It assesses how likely an issuer (the borrower) is to pay back investors (the lenders) and the
interest rate it may have to pay in return. They are conferred as letter grades, ranging from A at
the top to C or D at the bottom.
The three major credit rating agencies are Fitch Ratings, Moody's Investors Service and
S&P Global Ratings.
Obligations rated Baa2 are subject to moderate credit risk. They are considered medium
grade and as such may possess certain speculative characteristics.
In Moody's rating system, Baa3 is a relatively lower investment-grade rating. It signifies
a moderate credit risk for the issuer of the rated financial instrument or debt.

Therefore, option (b) is the correct answer.

Relevance: Moody's Investors Service has affirmed Reliance Industries Ltd's (RIL's) Baa2 domestic
long-term issuer rating and foreign currency senior unsecured rating.

Question 66. + 2.0 - 0.66

Which one of the following is not a component of the high-powered money of the Reserve Bank of
India (RBI)?

A. Cash in hand with the banks

B. Cash Reserve Ratio (CRR) deposits of banks

 C. Accounts of the International Monetary Fund

 D. Time deposits with the banking system

Question Analytics
4353 USERS 1114 USERS 3239 USERS 43.3 SECS

ATTEMPTED  SOLVED CORRECTLY  SOLVED INCORRECTLY  YOUR TIME

43.18 SECS 0.0 SECS

 AVG. SOLVING TIME  FASTEST SOLVING TIME

Explanation :

Reserve money (M0) is also known as central bank money, base money or high-powered
money. The bulk of the deposit liabilities, along with currency issued by the central bank, are
'monetary' in that they provide the base for credit creation as components of reserve money. It
plays a crucial role in the determination of monetary aggregates. Its components are:
Currency in Circulation: It comprises currency with the public and cash in hand
with the banks. The public’s demand for currency is determined by several factors such
as real income, price level, the opportunity cost of holding currency (i.e., the interest rate
on interest-bearing assets) and the availability of alternative instruments of transactions.
e.g., credit/debit cards, ATMs, cheque payments.
Bankers' Deposits with the Reserve Bank of India (RBI): A major component of the
reserve money is the bankers’ deposits with RBI, essentially arising out of the Cash
Reserve Ratio (CRR) prescribed for the banks.
"Other" Deposits with RBI: These include deposits of quasi-government and other
financial institutions including primary dealers, balances in the accounts of foreign
Central Banks and Governments, accounts of international agencies such as the
International Monetary Fund, etc.
M1 is a monetary aggregate used for assessing the money supply in the economy. It is referred
to as narrow money.
M1 = Currency with the Public + Demand Deposits with the Banking System +
‘Other’ Deposits with RBI
Thus, M1 = Currency with the Public + Current Deposits with the Banking System +
Demand Liabilities Portion of Savings Deposits with the Banking System + ‘Other’
Deposits with RBI
M2 = M1 + Time Liabilities Portion of Savings Deposits with the Banking System +
Certificates of Deposit issued by Banks + Term Deposits of residents with a contractual
maturity of up to and including one year with the Banking System

Thus, M2 = Currency with the Public + Current Deposits with the Banking System + Savings
Deposits with the Banking System + Certificates of Deposits issued by Banks + Term Deposits of
residents with a contractual maturity up to and including one year with the Banking System + ‘Other’
Deposits with RBI.

M3 = M2 + Term Deposits of residents with a contractual maturity of over one year with
the Banking System + Call/Term borrowings from ‘Non-depository’ financial
corporations by the Banking System.
‘Other’ deposits with RBI comprise mainly:
deposits of quasi-government and other financial institutions including primary dealers
balances in the accounts of foreign Central banks and Governments
accounts of international agencies such as the International Monetary Fund, etc.

Therefore, option (d) is the answer.

Knowledge Box
Government balances with the Reserve Bank of India (RBI) are considered non-
monetary because the State is treated as a part of the money-issuing sector along with the
central bank because of its ability to create money by fiat.
M0 is computed every week while M1 is calculated on a fortnightly basis.

Question 67. + 2.0 - 0.66

Which of the following securities qualify for the purpose of Statutory Liquidity Ratio (SLR)
maintained by banks in India?

1. State Development Loans


2. Bonds issued to the Food Corporation of India

3. Treasury Bills

4. Bank Recapitalisation Bonds


5. Cash Management Bills

Select the correct answer using the code given below:

A. 1, 2 and 4 only

 B. 3 and 5 only

 C. 1, 3 and 5 only

D. 1, 2, 3, 4 and 5

Question Analytics
3629 USERS 1280 USERS 2349 USERS 58.0 SECS

ATTEMPTED  SOLVED CORRECTLY  SOLVED INCORRECTLY  YOUR TIME

57.0 SECS 0.0 SECS

 AVG. SOLVING TIME  FASTEST SOLVING TIME

Explanation :

Statutory Liquidity Ratio (SLR) is a minimum percentage of deposits that a commercial


bank has to maintain in the form of liquid cash, gold or other securities. It is basically the
reserve requirement that banks are expected to keep before offering credit to customers. These
are not reserved with the Reserve Bank of India (RBI), but with banks themselves. The SLR
was prescribed by the Banking Regulation Act, 1949.
State Development Loans (SDLs) are market borrowings by State Governments. Each
State is allowed to issue securities up to a certain limit each year. SDLs qualify for SLR
status. So, point 1 is correct.
Under the market borrowing program, the Government of India issues, from time to time,
special bonds/securities to entities like Oil Marketing Companies, Fertilizer Companies, the
Food Corporation of India, etc. (popularly called oil bonds, fertiliser bonds and food bonds
respectively) as compensation to these companies in lieu of cash subsidies. These securities are
not eligible as SLR securities but are eligible as collateral for market repo transactions. So,
point 2 is not correct.
Treasury Bills or T-Bills are the instruments through which short term borrowings are
solicited by the Government of India. They are promissory notes issued at a discount to their
face value for a fixed time period e.g., 91 days, 182 days and 364 days. T-Bills qualify for SLR
securities. So, point 3 is correct.
The Government of India had issued Bank Recapitalisation Bonds to specific Public Sector
Banks in 2018. These securities are non-transferable and are not eligible investment in
pursuance of any statutory provisions or directions applicable to investment banks, including
the SLR. So, point 4 is not correct.
Cash Management Bills (CMB) have been introduced by the Reserve Bank of India to meet
temporary cash flow mismatches of the Government of India. CMBs are non-standard,
discounted instruments issued for maturities less than 91 days. Investment by banks in CMBs
is treated as an eligible investment in Government Securities for SLR purpose. So, point 5 is
correct.

Therefore, option (c) is the correct answer.


Relevance: According to the RBI, banks have increased their investments in Statutory Liquidity
Ratio (SLR) securities by 4.7 percent in the first quarter of FY24.

Question 68. + 2.0 - 0.66

In India, which one of the following authorities is responsible for the compilation of data for Balance
of Payments (BoP)?

A. Department of Economic Affairs

 B. Reserve Bank of India


C. Ministry of Commerce and Industry

 D. National Statistical Office

Question Analytics
4506 USERS 2272 USERS 2234 USERS 21.2 SECS

ATTEMPTED  SOLVED CORRECTLY  SOLVED INCORRECTLY  YOUR TIME

31.17 SECS 0.0 SECS

 AVG. SOLVING TIME  FASTEST SOLVING TIME

Explanation :

Balance of Payments (BoP) is a statistical statement summarising all the external transactions
(receipts and payments) on current and capital accounts in which a country is involved over
some time. As the BoP shows the total assets and obligations over a time period, it always
balances.
The compilation and dissemination of BoP data is the prime responsibility of the Reserve
Bank of India (RBI). In India, the compilation of BoP statistics is broadly consistent with the
guidelines contained in the BoP Manual, 5th Edition, of the International Monetary Fund
(BPM5). The format of the presentation of the BoP data for the period from 1990-91 is based
on the recommendations of the High-Level Committee under the chairmanship of Dr. C.
Rangarajan on Balance of Payments, 1993.

Therefore, option (b) is the correct answer.

Relevance: According to recent data released by the Reserve Bank of India (RBI), the Current
Account Deficit (CAD) was 3.8 per cent of Gross Domestic Product (GDP).

Question 69. + 2.0 - 0.66

Consider the following situations:


1. Increase in the Foreign Portfolio Investment (FPI) limit

2. Decrease in Dollar-Rupee swap


3. Increase in the External Commercial Borrowings (ECBs)

4. Restriction on the Foreign Capital Inflows

How many of the above are most likely to result in the appreciation of the Rupee?
A. Only one

  B. Only two

C. Only three

D. All four

Question Analytics
4124 USERS 2431 USERS 1693 USERS 56.0 SECS

ATTEMPTED  SOLVED CORRECTLY  SOLVED INCORRECTLY  YOUR TIME

69.96 SECS 0.0 SECS

 AVG. SOLVING TIME  FASTEST SOLVING TIME

Explanation :

Fall in the exchange rate i.e increase in the external value of domestic currency, due to additional
demand for home currency (or less supply of home currency) or less demand for (or more supply of)
foreign currency is called appreciation.

Increasing the Foreign Portfolio Investment (FPI) limit generally attracts more foreign
capital inflows, leading to increased demand for INR, potentially causing appreciation. When
the FPI limit is increased, it indicates greater confidence of foreign investors in the Indian
economy, leading to higher inflows of foreign funds. This can contribute to an increase in the
demand for Rupee, thereby causing its appreciation. So, point 1 is correct.
Decrease in Dollar-Rupee swap: It’s a forex tool whereby the central bank uses its
currency to buy another currency or vice versa. In a Dollar–Rupee buy/sell swap, the central
bank buys dollars (US dollars or USD) from banks in exchange for Indian Rupees (INR) and
immediately gets into an opposite deal with banks promising to sell dollars at a later date. In a
dollar–rupee sell/buy swap it sells USD in exchange for INR and promises to buy dollars from
banks after some years.
When there is decrease in the dollar-rupee swap, it implies less demand for rupee
globally which results in the depreciation of the domestic currency. Thus, the rupee will
depreciate. So, point 2 is not correct.
External Commercial Borrowing (ECB) is a type of loan in foreign currencies, made by non-
resident lenders. Thus, easing conditions of the ECB helps in receiving more loans in foreign
currencies. This would increase the inflow of forex, leading to rupee appreciation. So,
point 3 is correct.
Capital inflows result in a buildup of foreign exchange reserves. As these reserves are used
to buy domestic currency, the domestic monetary base expands without a corresponding
increase in production: too much money begins to chase too few goods and services. This
creates appreciation of the domestic currency. Restricting the foreign capital inflows will
inverse the situation and thus lead to depreciation of the rupee. So, point 4 is not correct.

So, only two of the above are most likely to result in the appreciation of the Rupee.

Therefore, option (b) is the correct answer.

Question 70. + 2.0 - 0.66

The Catastrophe Containment and Relief Trust (CCRT) is the:

World Health Organization’s (WHO) provision for medical aid during public health
A.
emergency

International Monetary Fund’s (IMF) debt relief to the countries affected by natural
 B.
disasters

Regional Anti-Terrorist Structure’s (RATS) framework to contain terrorism in member


 C.
countries

United Nations Peacekeeping Forces’ relief services to communities affected in conflict


D.
zones

Question Analytics
3434 USERS 1884 USERS 1550 USERS 30.7 SECS

ATTEMPTED  SOLVED CORRECTLY  SOLVED INCORRECTLY  YOUR TIME

33.37 SECS 0.0 SECS

 AVG. SOLVING TIME  FASTEST SOLVING TIME

Explanation :

The Catastrophe Containment and Relief Trust (CCRT) was established by the
International Monetary Fund (IMF) to provide grants for debt relief to the poorest and
most vulnerable countries affected by catastrophic natural disasters or public health crises,
enabling them to meet their debt service obligations to the IMF. This relief frees up resources
of these countries, helping them meet exceptional balance of payments needs created by the
disaster and to pay for containment and recovery.
CCRT assistance is available to countries if they are eligible to borrow from the IMF’s
Poverty Reduction and Growth Trust (PRGT) and their per-capita income is less than the
International Development Association (IDA) operational cutoff.
Countries qualify for relief if a natural disaster has directly affected at least one-third of
the population, is estimated to have destroyed more than a quarter of the country’s productive
capacity or has caused damage deemed to exceed 100 percent of GDP.
Eligibility for public health disaster relief covers life-threatening epidemics and
pandemics that has affected several areas of their country, has the potential to spread or
is already spreading to other countries, and is causing significant economic disruption.

Therefore, option (b) is the correct answer.


Relevance: Managing director of the International Monetary Fund (IMF) has identified domestic
resource mobilisation to accelerate decarbonisation.

Question 71. + 2.0 - 0.66

Which of the following can be used as trademarks under the Trade Marks Act, 1999?

1. Letters or numerals

2. Sound marks
3. Combination of colours

4. Name of chemical element


5. Shape of goods

Select the correct answer using the code given below:

A. 1, 2 and 4 only

B. 3, 4 and 5 only

 C. 1, 2, 3 and 5 only

D. 1, 2, 3, 4 and 5

Question Analytics
3486 USERS 1585 USERS 1901 USERS 52.6 SECS

ATTEMPTED  SOLVED CORRECTLY  SOLVED INCORRECTLY  YOUR TIME

44.33 SECS 0.0 SECS

 AVG. SOLVING TIME  FASTEST SOLVING TIME


Explanation :

The Trade Marks Act, 1999 adheres to Trade-Related Aspects of Intellectual Property
Rights (TRIPS), 1994.
Types of trademarks that may be registered in India:
Any name (including personal or surname of the applicant or predecessor in business or
the signature of the person), which is not unusual for trade to adopt as a mark.
An invented word or any arbitrary dictionary word or words, not being directly
descriptive of the character or quality of the goods/service.
Letters or numerals or any combination thereof. So, point 1 is correct.
The right to proprietorship of a trademark may be acquired by either registration under
the Act or by use in relation to particular goods or service.
Devices, including fancy devices or symbols
Monograms
Combination of colours or even a single color in combination with a word or device.
So, point 3 is correct.
Shape of goods or their packaging. So, point 5 is correct.
Marks constituting a 3- dimensional sign.
Sound marks when represented in conventional notation or described in words by being
graphically represented. So, point 2 is correct.
The Trade Marks Act, 1999 prohibits the registration of trade marks that include a
frequently used and accepted name of any single chemical element or chemical compound
in relation to a chemical product or preparation. A word that has been declared as an
International Non-Proprietary Name (INN) by WHO (World Health Organization) or that is
deceptively similar to such a name as informed by the Registrar is not permitted to be
registered. So, point 4 is not correct.

Therefore, option (c) is the correct answer.

Knowledge Box

Signature Trademarks registered in India:

While the registration of a trademark is not required by law, it is recommended since


unregistered trademarks receive little protection. Celebrities often register their signatures,
names, initials etc.
Question 72. + 2.0 - 0.66

Consider the following fish species:


1. Guppy

2. Aplocheilus panchax

3. Oryzias melastigma
4. Gambusia affinis

How many of the above can be used to control the mosquito spread?

A. Only one

B. Only two

C. Only three

 D. All four

Question Analytics
2145 USERS 1045 USERS 1100 USERS 37.8 SECS

ATTEMPTED  SOLVED CORRECTLY  SOLVED INCORRECTLY  YOUR TIME

16.3 SECS 0.0 SECS

 AVG. SOLVING TIME  FASTEST SOLVING TIME

Explanation :

Several species of both indigenous and exotic fish are used to control the mosquito population
because of their larvivorous potential.

Poecilia reticulata: It is also called ‘Guppy’ and is an exotic fish introduced in India. It
cannot survive in cold water below 10°C. It is a surface feeder and a single fish eats about 80
to 100 mosquito larvae in 24 hours. So, point 1 is correct.
Aplocheilus panchax: It is an indigenous variety and widely distributed in Bengal, Bihar,
Orissa, Assam, Punjab, Uttar Pradesh, Madhya Pradesh and Rajasthan. It is a potential
larvivorous fish in controlling several vector species in different types of natural and man-
made habitats. It is used for filariasis vector control and is effective on both anophelines and
culicines. So, point 2 is correct.
Oryzias melastigma: It is an indigenous fish and is widely distributed in Bengal, Odisha and
Tamil Nadu. It is a surface feeder found in both still and running waters. It is suitable for open
shallow water stretches especially in rice fields for control of mosquitoes causing Japanese B
encephalitis. So, point 3 is correct.
Gambusia affinis: It is an exotic species and a surface feeder, hence it is suitable for feeding
on both anophelines and culicines. The fish has been used extensively in different parts of the
country for the control of Anopheles stephensi breeding particularly in urban areas. So, point
4 is correct.
Some other species of larvivorous fish include Aphanius dispar, Colisa fasciatus, Chanda
nama, Carassius auratus, Xenentodon cancila etc.

So, all four of the above can be used to control the mosquito spread.
Therefore, option (d) is the correct answer.

Relevance: The Andhra Pradesh Government has released approximately 10 million Gambusia fish
into the state’s water bodies to combat mosquito-borne diseases like malaria and dengue.

Question 73. + 2.0 - 0.66

Consider the following statements with reference to the nomenclature of the tropical cyclones in the
North Indian Ocean:

1. The World Meteorological Organisation (WMO) is responsible for naming all tropical cyclones.
2. The names of tropical cyclones once used, cannot be used again.

3. The name of the cyclone should not exceed eight letters.

How many of the above statements are correct?

A. Only one

 B. Only two

C. All three

D. None

Question Analytics
4148 USERS 2063 USERS 2085 USERS 19.5 SECS

ATTEMPTED  SOLVED CORRECTLY  SOLVED INCORRECTLY  YOUR TIME

41.75 SECS 0.0 SECS

 AVG. SOLVING TIME  FASTEST SOLVING TIME

Explanation :
The Regional Specialized Meteorological Centre (RSMC) New Delhi Tropical Cyclone
Center is responsible for naming the tropical cyclones that have formed over the Bay of
Bengal and the Arabian Sea when they have reached the relevant intensity. So, statement 1 is
not correct.
Criteria for selection of tropical cyclones names by the Panel on Tropical Cyclones (PTC)
member countries:
The proposed name should be neutral to (a) politics and political figures (b) religious
believes, (c) cultures and (d) gender
Name should be chosen in such a way that it does not hurt the sentiments of any group
of population over the globe
It should not be very rude and cruel in nature
It should be short, easy to pronounce and should not be offensive to any member
The maximum length of the name will be eight letters. So, statement 3 is correct.
The proposed name should be provided along with its pronunciation, meaning and voice
over
The Panel reserves the right to reject any name, if any of the criteria above is not
satisfied.
The finalised names may also be reviewed during the course of time of implementation
with the approval of PTC in its annual session, in case any reasonable objection is raised
by any member
The names of tropical cyclones over the North Indian Ocean will not be repeated, once
used it will cease to be used again. The name should be new. It should not be there in
the already existing list of any of the RSMCs worldwide including RSMC New Delhi.
So, statement 2 is correct.

So, only two of the above statements are correct.

Therefore, option (b) is the correct answer.


Relevance: Cyclone Michaung made landfall over Nellore in Andhra Pradesh as a super-cyclonic
storm.

Question 74. + 2.0 - 0.66

‘Bromeliads’, sometimes seen in the news, are:

 A. carnivorous plants that capture insects for nutrition


 B. plants that can store water in their leaves

C. frost-resistant plants that can thrive in cold climates

D. plants that have adapted to grow in saline conditions

Question Analytics
2863 USERS 1048 USERS 1815 USERS 13.4 SECS

ATTEMPTED  SOLVED CORRECTLY  SOLVED INCORRECTLY  YOUR TIME

29.05 SECS 0.0 SECS

 AVG. SOLVING TIME  FASTEST SOLVING TIME

Explanation :

Bromeliads are special plants that store water in their leaves. They are native to tropical
North and South America. The commercial pineapple (Ananas comosus) is native to
southern Brazil and Paraguay.
They typically have bright red, orange, purple, or blue flowers, and can grow in several
different ways: they can be terrestrial, saxicolous (growing on rocks) or epiphytic (growing on
other plants and trees).
Many bromeliads are tropical epiphytes. They have a special form of photosynthesis that uses
a variation of the more usual biochemical pathways to allow them to use water more
efficiently.
They have stiff, overlapping leaves which hold rainfall like buckets. Leaves and debris fall
into these reservoirs and help algae and other single-celled organisms to grow, which in turn
feed mosquitoes, insect larvae, and other organisms. The bromeliad is like a small ecosystem
in itself. Animals such as tree frogs, snails, flatworms, tiny crabs, and salamanders might spend
their entire lives inside them. Animals like frogs use these pockets of water to lay their eggs.

Therefore, option (b) is the correct answer.


Relevance: As per a recent study, Bromeliads are at a greater threat due to rapid environmental
change.

Question 75. + 2.0 - 0.66

Consider the following species:

1. Shama
2. Dhyal

3. Pied wagtail
4. Indian robin

How many of the above are songbirds found in India?

A. Only one

B. Only two

C. Only three

 D. All four

Question Analytics
1754 USERS 829 USERS 925 USERS 4.4 SECS

ATTEMPTED  SOLVED CORRECTLY  SOLVED INCORRECTLY  YOUR TIME

14.57 SECS 0.0 SECS

 AVG. SOLVING TIME  FASTEST SOLVING TIME

Explanation :

India is home to some songbirds capable of competing with the best. There are numerous
songbirds in India.
The shama (Cittocincla macrura) is one of the well known songbirds. It is not likely to be
heard in any large town. It lives among shady hills and the great forests of India. So, point 1 is
correct.
A fine songbird, nearly related to the shama, and found in nearly all parts of India, is the
magpie robin or dhyal (Copsychus saularis). It has the confiding habits of the robin, the
bright colouring of the magpie, and the voice of the canary. It is nearly always found near
human habitations. So, point 2 is correct.
The small pied wagtail is the most attractive of birds that nest around human habitations and
brings joy with its quiet, sweet voice throughout the year. So, point 3 is correct.
The Indian robin is a singing bird found all over India, but robins south of the Godavari River
differ slightly from those which dwell in Northern India. So, point 4 is correct.

So, all four of the above are songbirds found in India.

Therefore, option (d) is the correct answer.


Relevance: The UN report stated that nearly half of the world's migratory species including
songbirds are on the decline.

Question 76. + 2.0 - 0.66

Which one of the following species is recently seen in the news in context of efforts taken to preserve
the Amazon rainforest?

 A. Stingless bee

B. Hoverfly

C. Fig Wasp

D. Elephant Hawk Moth

Question Analytics
2115 USERS 807 USERS 1308 USERS 25.8 SECS

ATTEMPTED  SOLVED CORRECTLY  SOLVED INCORRECTLY  YOUR TIME

25.14 SECS 0.0 SECS

 AVG. SOLVING TIME  FASTEST SOLVING TIME

Explanation :

A recent mission has commenced in Peru aimed at preserving the Stingless Bee, which is
indigenous to the tropics. These pollinators are playing a pivotal role in one of the most
recent endeavours to safeguard the Amazon rainforest.
Stingless bees are native to the Amazon, unlike the more familiar but invasive honey bees
from Africa and Europe that have spread through the Americas. The most obvious difference,
perhaps, is that stingless bees don’t sting. Their honey, which is runny enough to be drunk like
a liquid and is said to have a citrusy aftertaste, is used by many Indigenous Peruvians as a
natural medicine.

Therefore, option (a) is the correct answer.

Relevance: Recently, in Peru, a Mission started to Save the Stingless Bee - Native to the tropics,
these pollinators are taking a lead role in one of the latest efforts to conserve the Amazon rainforest.

Question 77. + 2.0 - 0.66


Consider the following statements with reference to bats:

1. Some bat species feed on insects like moths.

2. Some bat species use echolocation to navigate dark caves.


3. Some bat species hibernate to survive harsh winters.

4. Some bat species can be found in polar regions.


How many of the above statements are correct?

A. Only one

B. Only two

  C. Only three

D. All four

Question Analytics
4181 USERS 1278 USERS 2903 USERS 23.5 SECS

ATTEMPTED  SOLVED CORRECTLY  SOLVED INCORRECTLY  YOUR TIME

24.04 SECS 0.0 SECS

 AVG. SOLVING TIME  FASTEST SOLVING TIME

Explanation :

Bats are the only mammals in the world that can fly. There are two main types of bats:
microbats and megabats. Most bats are microbats, which eat insects like moths that come
out at night. Vampire bats are the only species of microbats that feed on blood rather than
insects. So, statement 1 is correct.
To navigate dark caves and hunt after dark, microbats rely on echolocation, a system that
allows them to locate objects using sound waves. They echolocate by making a high-pitched
sound that travels until it hits an object and bounces back to them. This echo tells them an
object’s size and how far away it is. So, statement 2 is correct.
In contrast, megabats live in the tropics and eat fruit, nectar, and pollen. They have
larger eyes and a stronger sense of smell than microbats but have smaller ears because
they don’t echolocate.
Bears and bats are the two most well-known hibernators to survive harsh winters. Some bat
species like the spotted bat survive by migrating in search of food to warmer areas when it gets
chilly. So, statement 3 is correct.
Bats can be found on nearly every part of the planet except in extreme deserts and polar
regions. They spend their daylight hours hiding in roosts around the tropics, dense forests, and
wetlands. Roosts are where bats go to rest, usually in cracks and crevices that keep them
hidden and protected. The most common roosts are existing structures such as caves, tree
hollows and old buildings. So, statement 4 is not correct.
Vampire bat saliva includes an anticoagulant (a substance that stops blood clots from forming)
that might be utilised to treat human heart ailments.

So, only three of the above statements are correct.


Therefore, option (c) is the correct answer.

Relevance: Recently, a research was published related to the immune response of bats.

Question 78. + 2.0 - 0.66

Consider the following species:


1. Dhole (wild dog)

2. Sloth bear
3. Mouse deer

4. Lion-tailed macaque

5. Wild ass
How many of the above are most likely to be observed if you visit the Bandipur National Park?

A. Only two

 B. Only three

 C. Only four

D. All five

Question Analytics
3250 USERS 1153 USERS 2097 USERS 19.8 SECS

ATTEMPTED  SOLVED CORRECTLY  SOLVED INCORRECTLY  YOUR TIME

29.96 SECS 0.0 SECS

 AVG. SOLVING TIME  FASTEST SOLVING TIME

Explanation :
Bandipur National Park is located in Karnataka in the Western Ghats. It is an important
part of the Nilgiri Biosphere Reserve that constitutes Karnataka’s Rajiv Gandhi National
Park (Nagarahole) to its Northwest, Tamil Nadu’s Mudumalai Wildlife Sanctuary to its South,
and Kerala’s Wayanad Wildlife Sanctuary to its Southwest.
Several endangered species such as Asian Wild Elephants, Tigers, sloth bears, gaurs, dhole
(wild dog) Indian rock pythons, jackals, muggers, and four-horned antelopes can be
spotted in this national park. Bandipur also shelters sambar, mouse deer, chital and the rare
flying lizard. So, points 1, 2 and 3 are correct.
It is also home to several primate species, including the endangered Nilgiri langur and the lion-
tailed macaque. So, point 4 is correct.
Wild asses are found in the Rann of Kutch of Gujarat. So, point 5 is not correct.

So, only four of the above are most likely to be observed if you visit the Bandipur National
Park.
Therefore, option (c) is the correct answer.

Relevance: Bandipur National Park, which is home to more than 1100 elephants as per the last
estimation, is set to get a task force to deal with conflict situations.

Question 79. + 2.0 - 0.66

Consider the following statements:


1. All transgenic organisms are Genetically Modified Organisms (GMOs), but not all GMOs are
transgenic.
2. The Ministry of Environment, Forest and Climate Change is the final authority to approve the
testing of transgenic crops in India.

3. The import of GMOs for food and feed is regulated by the Seeds Act of 1966.

How many of the above statements are correct?

 A. Only one

B. Only two

C. All three

D. None

Question Analytics
3410 USERS 1051 USERS 2359 USERS 5.9 SECS
ATTEMPTED  SOLVED CORRECTLY  SOLVED INCORRECTLY  YOUR TIME

38.75 SECS 0.0 SECS

 AVG. SOLVING TIME  FASTEST SOLVING TIME

Explanation :

Transgenic refers to an organism or cell whose genome has been altered by the introduction
of one or more foreign DNA sequences from another species by artificial means. Transgenic
organisms are generated in the laboratory for research purposes.
Genetically Modified Organism (GMO) and transgenic organism are two terms often used
interchangeably. Both types of organisms have an altered genome that has been modified
artificially. However, there is a slight difference between GMO and transgenic organisms.
Although both have altered genomes, a transgenic organism is a GMO containing a DNA
sequence or a gene from a different species. Thus, all transgenic organisms are GMOs, but
not all GMOs are transgenic. So, statement 1 is correct.
Transgenic animals have been generated to understand normal physiological processes such
as metabolism and blood cell production. They have been used to model human diseases and
to develop new treatments. Transgenic crops have been produced that are resistant to
infectious diseases and grow faster.
Agriculture is a State subject as per the Seventh Schedule of the Constitution of India. It
implies that companies interested in testing transgenic crops need approvals from the States
to conduct such tests. Only Haryana permitted such tests. So, statement 2 is not correct.
The import of Genetically Modified Organisms for research & development, food, feed,
processing in Bulk and for environmental release is governed by the provisions of the
Environment Protection Act, 1986 and Rules 1989. Environmental release or field
application will be allowed only with the approval of the Genetic Engineering Approval
Committee (GEAC). So, statement 3 is not correct.

So, only one of the above statements is correct.


Therefore, option (a) is the correct answer.

Knowledge Box

Genetic Engineering Appraisal Committee (GEAC):

It functions under the Union Ministry of Environment, Forest and Climate Change and is
responsible for the appraisal of activities involving large-scale use of hazardous
microorganisms and recombinants in research and industrial production from the
environmental angle.
It is also responsible for the appraisal of proposals relating to the release of Genetically
Engineered (GE) organisms and products into the environment including experimental field
trials.

Relevance: Recently, a few States have deferred a proposal to test a new kind of transgenic cotton
seed that makes it resistant to pink bollworms.

Question 80. + 2.0 - 0.66

Consider the following activities:

1. Injection of carbonic acid into ultramafic rocks

2. Coal gasification
3. Carbonation of concrete

4. Ocean alkalinization
Which of the above are often considered for Carbon Mineralization?

A. 1 and 2 only

  B. 1, 3 and 4 only

C. 3 and 4 only

D. 1, 2, 3 and 4

Question Analytics
2901 USERS 1134 USERS 1767 USERS 59.0 SECS

ATTEMPTED  SOLVED CORRECTLY  SOLVED INCORRECTLY  YOUR TIME

38.69 SECS 0.0 SECS

 AVG. SOLVING TIME  FASTEST SOLVING TIME

Explanation :

‘Carbon Mineralization’ is a natural phenomenon wherein atmospheric carbon dioxide (CO2)


reacts with certain minerals within rocks, efficiently removing CO2 from the atmosphere. This
process securely stores the carbon in the form of carbonates. This process occurs naturally over
hundreds or thousands of years, but can be sped up to provide a permanent solution for Carbon
Sequestration in order to mitigate global warming as follows:
Carbonic acid (H2CO3) injection into Mafic or Ultramafic rocks involves dissolving
concentrated CO2 in water to create carbonated water or carbonic acid (H2CO3).
The resulting solution is injected into wells drilled into mafic or ultramafic rocks, known
for their high reactivity with CO2. Within these rocks, carbonate minerals form as a
result of the reaction, effectively capturing and sequestering the CO2. Subsequently, the
water depleted of CO2 is brought back to the Earth's surface. So, point 1 is correct.
Carbonation of concrete is the chemical reaction between carbon dioxide (CO2) in the
air and calcium hydroxide and hydrated calcium silicate in the concrete to give mainly
carbonates. Injecting captured CO2 into concrete before its hardening forms calcium
carbonate, enhancing concrete strength and reducing cement usage. Additionally, this
process securely stores CO2, contributing to long-term Carbon Sequestration. So, point
3 is correct.
Ocean alkalinization is a carbon removal method where alkaline substances are
introduced into seawater to boost its natural carbon absorption capacity. These
substances, like olivine minerals or industrial byproducts like lime, chemically react with
dissolved carbon dioxide (CO2) in the ocean, converting it into stable bicarbonate and
carbonate molecules. This process enhances the ocean's ability to absorb CO2 from
the atmosphere, aiding in restoring equilibrium. So, point 4 is correct.
Coal Gasification is a process in which coal is partially oxidised by air, oxygen, steam or
carbon dioxide under controlled conditions to produce a fuel gas. The hot fuel gas is cooled in
heat exchangers, with the production of steam, and cleaned before combustion in a gas turbine.
It is not considered for Carbon Mineralization. So, point 2 is not correct.

Therefore, option (b) is the correct answer.


Relevance: The Union Cabinet has approved India’s updated Nationally Determined Contribution
(NDC) to be communicated to the United Nations Framework Convention on Climate Change
(UNFCCC).

Question 81. + 2.0 - 0.66

In context of history of medieval India, the officers ‘qazi- ul- quzat, tui-begis and Mir- adl’ were
associated with:
A. intelligence and espionage

B. revenue collection

 C. administration of justice

D. foreign affairs

Question Analytics
3808 USERS 3008 USERS 800 USERS 42.7 SECS

ATTEMPTED  SOLVED CORRECTLY  SOLVED INCORRECTLY  YOUR TIME

23.78 SECS 0.0 SECS

 AVG. SOLVING TIME  FASTEST SOLVING TIME

Explanation :

Qazi-ul-qazat was the chief official with a position next to the emperor. He decided cases in
accordance with the Islamic law and also appointed Qazis in different parts of empires. The
king appointed the chief qazi sadr who possessed the powers of judge and had the power of
appointing subordinate qazis in the dominations, though the king’s sanction was necessary in
all such appointments.
The king also exercised his power to appoint more than one judge in a city and their duties
were accordingly defined. These were the qazis and mir adls. In all big cities and towns the
two existed side by side.
Qazi –i- mamalik or Qazi- ul- quzat was the name given to the Chief Judge of the sultanate.
The secular judge was called Mir- adl during the Mughal era. He represented the judge on
account of the Emperor. It was his sole duty and responsibility to implement Qazi’s decisions
and he was also called to make unprejudiced and individual inquiries.
The two officers were appointed by Akbar called Tui-begis. It was their duty to look into
obedience to the law and a very affordable amount of money was fixed as their remuneration
or fee.

Therefore, option (c) is the correct answer.

Knowledge Box

Mir Saman or Khan-i- Saman was the minister of the royal Family and looked after royal
buildings, roads, parks, karkhanas etc. He was incharge of the imperial household including
the supply of all the provisions and articles for the use of inmates of the haram or the female
apartments.
Amin, Qanungo were appointed as revenue officials.
Mir Bakshi headed the military department, nobility, information and intelligence agencies.

Question 82. + 2.0 - 0.66

Consider the following:

1. Marco Polo
2. Al-Biruni

3. Nicolo De Conti

4. Ibn Battuta
Which one of the following is the correct chronological order of the above foreign travellers' visit to
India?

A. 2–3–1–4

 B. 1–2–4–3

 C. 2–1–4–3

D. 4–2–3–1

Question Analytics
3499 USERS 1883 USERS 1616 USERS 110.4 SECS

ATTEMPTED  SOLVED CORRECTLY  SOLVED INCORRECTLY  YOUR TIME

42.1 SECS 0.0 SECS

 AVG. SOLVING TIME  FASTEST SOLVING TIME

Explanation :

Al-Biruni: He was born in Khwarizm (Uzbekistan).. He was a scholar who was well versed in
Syriac, Arabic, Persian, Hebrew and Sanskrit. In 1017, Sultan Mahmud Ghazni invaded
Khwarizm and took many scholars including Al-Biruni to his capital, Ghazni. Owing to the
inclusion of Punjab in the Ghaznavid Empire, Al-Biruni travelled widely in Punjab and other
Northern parts of India from 1024- 1030 AD. He translated several Sanskrit works, including
Patanjali’s work on Grammar and also authored ‘Kitab-ul-Hind’.
Marco Polo: He was born in the republic of Venice. He was an Italian merchant, adventurer
and writer who wrote “The Travels of Marco Polo”, an account of his travels from one country
to another. He visited the Kakatiya kingdom in around 1289 AD during the reign of Queen
Rudramadevi.
Ibn Battuta: He was a Moroccan traveller who was born in Tangier. Throughout his life, he
travelled extensively in Syria, Iraq, Persia, Yemen, Oman and a few trading ports on the coast
of East Africa. He came to India in 1332–33 AD during the reign of Muhammad Bin
Tughlaq who was impressed by his scholarship, and appointed him as “Qazi” or judge of
Delhi. He wrote a book of travels in Arabic called ‘Rihla’.
Nicolo De Conti: He was an Italian merchant and explorer. He crossed the Arabian Sea to
Cambay in Gujarat and entered India in 1420-1421 AD. He travelled to the Vijayanagara
kingdom during the reign of Deva Raya I. He referred to the Telugu language as “Italian of
the East”. He described South-East Asia as “overtaking all other regions in terms of wealth,
culture and magnificence”.

So, the correct chronological order of the above mentioned foreign travellers visit to India is
Al-Biruni - Marco Polo - Ibn Battuta - Nicolo De Conti.

Therefore, option (c) is the correct answer.

Question 83. + 2.0 - 0.66

Consider the following statements with reference to Adi Shankaracharya:

1. He proposed “Advaita Vedanta” to promote non-dualism of reality.

2. He set up four cardinal mathas in four corners of India.


3. He authored the books “Gita Bhasya” and “Vedantasara”.

4. He was a contemporary of Ramanujacharya.


How many of the above statements are correct?

A. Only one

 B. Only two

C. Only three

D. All four

Question Analytics
3567 USERS 1042 USERS 2525 USERS 27.4 SECS
ATTEMPTED  SOLVED CORRECTLY  SOLVED INCORRECTLY  YOUR TIME

30.07 SECS 0.0 SECS

 AVG. SOLVING TIME  FASTEST SOLVING TIME

Explanation :

Adi Shankaracharya is one of the most important figures in Hinduism and one of the
foremost proponents of Advaita Vedanta. He was born in Kerala in the eighth century CE.
He had set up four cardinal mathas at Badrinath, Sringeri, Puri and Dwarka in the North,
South, East and West of India respectively. So, statement 2 is correct.
Advaita Vedanta is a school of Hindu philosophy and spiritual discipline. It articulates an
ontological position of radical nondualism. It posits that all that we perceive is ultimately
illusory (maya) and that the principle of Brahman is the only true reality of all things,
transcending empirical plurality. The fundamental idea of Advaita Vedanta lies in the unity of
Atman or individual consciousness and Brahman or the ultimate reality. So, statement 1
is correct.
He is also identified as the author of multiple works, including celebrated commentaries
(or bhashyas) on 10 Upanishads, the Brahma Sutra and the Bhagavad Gita and wrote
books like Upadesh Shastri, Vivek Chudamani and Bhaja Govindum Stotra.
Ramanujacharya lived in the eleventh and twelfth century CE. He was one of the most
important exponents of the Sri Vaishnavism tradition within Hinduism. His philosophical
foundations for devotionals were influential to the Bhakti movement. His Vishishtadvaita
(qualified non-dualism) philosophy has competed with the Dvaita (theistic dualism)
philosophy of Madhvacharya, and the Advaita (non-dualism) philosophy of Adi
Shankaracharya. He wrote books such as Sribhashya, Vedanta Dip, Gita Bhasya and
Vedantasara. He and Adi Shankaracharya were not contemporaries. So, statements 3 and
4 are not correct.

So, only two of the above statements are correct.


Therefore, option (b) is the correct answer.

Knowledge Box

Shankaracharya or ‘teacher of the way of Shankara’, is a religious title used by the heads
of the four cardinal mathas or peeths believed to have been established by Adi Shankara.
According to tradition, they are religious teachers who belong to a line of teachers going back
to Adi Shankara himself.

Relevance: Recently, the Shankaracharyas of four mathas in India refused to attend the inauguration
of Ram Mandir.

Question 84. + 2.0 - 0.66

Doli-Puran, a historical ballad composed by Narrotam Das in Assamese, narrates events related to
which one of the following?

A. Paharias' Rebellion

B. Chuar Uprising

C. Sanyasi Revolt

 D. Patharughat Uprising

Question Analytics
2506 USERS 1313 USERS 1193 USERS 12.5 SECS

ATTEMPTED  SOLVED CORRECTLY  SOLVED INCORRECTLY  YOUR TIME

24.94 SECS 0.0 SECS

 AVG. SOLVING TIME  FASTEST SOLVING TIME

Explanation :

Doli-Puran is a historical ballad in Assamese. Composed by Narrottam Das, it narrates


events related to the Patharughat Peasant Resistance of 1894 that took place in Assam. On
28 January 1894, more than a hundred peasants fell to the bullets of the British army in
Darrang. The unarmed peasants were peacefully protesting against the increase in land
revenue levied by the British when the military opened fire.
The book, Doli Puran, was recovered in Udalguri district and was first published in 1949.
Narottam was an eyewitness to the uprising and described the events in his song Doli
(earth/soil). The unarmed peasants are said to have thrown clods of soil at the British police
who were firing at them.

Therefore, option (d) is the correct answer.

Question 85. + 2.0 - 0.66


In colonial India, the Delhi Durbars were organised by the Viceroys to:

  A. mark the coronation of the Monarch of Great Britain

B. celebrate the renewal of trade licence of the East India Company

C. honour friendly Princely States in British India

D. mark the transfer of Viceroyship to the newly appointed officer

Question Analytics
4297 USERS 3382 USERS 915 USERS 67.2 SECS

ATTEMPTED  SOLVED CORRECTLY  SOLVED INCORRECTLY  YOUR TIME

26.82 SECS 0.0 SECS

 AVG. SOLVING TIME  FASTEST SOLVING TIME

Explanation :

The Delhi Durbars were grand events organised by the Viceroys to mark the coronations of
Emperors or Empresses of Great Britain. Hence, these were also known as the Coronation
Durbars. Durbar, which means a 'court of a ruler' in Persian, was adopted by the British from
the Mughals. The idea was to come across to the people of India as the 'heirs of the Mughals'.

Therefore, option (a) is the correct answer.

Knowledge Box

Delhi Durbar 1877: Queen Victoria in addition to her title of 'Queen of Great Britain and
Ireland' assumed the title of 'Empress of India'.
Delhi Durbar 1903: It marked the succession of Edward VII.
Delhi Durbar 1911: It marked the succession of King George V. It was historic for two
reasons; one it was the only one attended by the Emperor himself and second, this was
where the shifting of the Imperial Capital from Calcutta to Delhi was announced.

Question 86. + 2.0 - 0.66

Consider the following:

1. Publication of Al-Hilal and Comrade newspapers

2. Visit of the Prince of Wales


3. Protest by Khudai Khidmatgars

4. Establishment of parallel governments

5. March on the Dharasana Salt Works


How many of the above events took place during the Civil Disobedience Movement (CDM)?

A. Only two

 B. Only three

C. Only four

 D. All five

Question Analytics
2898 USERS 1422 USERS 1476 USERS 39.0 SECS

ATTEMPTED  SOLVED CORRECTLY  SOLVED INCORRECTLY  YOUR TIME

36.25 SECS 0.0 SECS

 AVG. SOLVING TIME  FASTEST SOLVING TIME

Explanation :

Inspired by Mahatma Gandhi’s march to Dandi, the Civil Disobedience Movement (CDM) of 1930
in India stretched to every corner of the country, touching even the smallest districts.

Both Al Hilal (1912-1914) and Comrade (1911-1914) were prominent Urdu weekly
newspapers that played a significant role in shaping Indian public opinion during the early 20th
century. However, these were not associated with the CDM. So, point 1 is not correct.
The visit of the Prince of Wales began on 17 November 1921 during the Non-Cooperation
Movement and not CDM. The day the Prince landed in Bombay was observed as a day of
hartal all over the country. He was greeted with empty streets and downed shutters wherever he
went. So, point 2 is not correct.
Khudai Khidmatgars participated actively in the CDM in 1930-31. They played a
significant role in the movement, particularly in the North-West Frontier Province (NWFP).
So, point 3 is correct.
Textile workers of Solapur in Maharashtra went on a strike during CDM and along with
other residents burnt liquor shops and other symbols of Government authority such as railway
stations, police stations, municipal buildings, law courts, etc. The activists established a virtual
parallel government which could only be dislodged with martial law. So, point 4 is correct.
During the CDM, On May 21, 1930, Sarojini Naidu, Imam Sahib and Manilal (Gandhi’s
son) took up the unfinished task of leading a raid on the Dharasana Salt Works. The
unarmed and peaceful crowd was met with a brutal lathi-charge. This new form of salt
satyagraha was eagerly adopted by people in Wadala (Bombay), Karnataka (Sanikatta Salt
Works), Andhra, Midnapore, Balasore, Puri and Cuttack. So, point 5 is correct.

So, only three of the above events took place during the Civil Disobedience Movement (CDM).

Therefore, option (b) is the correct answer.

Question 87. + 2.0 - 0.66

Consider the following trees:


1. Mahogany

2. Ebony
3. Khejri

How many of the above are found in the Western Ghats?

A. Only one

 B. Only two

 C. All three

D. None

Question Analytics
3863 USERS 2336 USERS 1527 USERS 9.6 SECS

ATTEMPTED  SOLVED CORRECTLY  SOLVED INCORRECTLY  YOUR TIME

18.26 SECS 0.0 SECS

 AVG. SOLVING TIME  FASTEST SOLVING TIME

Explanation :

Tropical Evergreen Forest: These forests are found in the western slope of the Western
Ghats, hills of the northeastern region and the Andaman and Nicobar Islands. They are
found in warm and humid areas with an annual precipitation of over 200 cm and a mean annual
temperature above 220 celsius. Species found in these forests include rosewood, mahogany,
aini, ebony, etc. So, points 1 and 2 are correct.
Tropical Thorn Forests: These occur in the areas which receive rainfall less than 50 cm.
These consist of a variety of grasses and shrubs. It includes semi-arid areas of south west
Punjab, Haryana, Rajasthan, Gujarat, Madhya Pradesh and Uttar Pradesh. Important species
found are babool, ber, and wild date palm, khair, neem, khejri, palas, etc. So, point 3 is not
correct.
Tropical Deciduous Forests: These are the most widespread forests in India. They are also
called the monsoon forests. They spread over regions which receive rainfall between 70-200
cm. Tendu, palas, amaltas, bel, khair, axlewood, etc. are the common trees of these forests.
Montane Forests: In mountainous areas, the decrease in temperature with increasing altitude
leads to a corresponding change in natural vegetation. In the Himalayan region, between
1,500-1,750 m, Chir Pine is a very useful commercial tree. Deodar, a highly valued endemic
species, grows mainly in the western part of the Himalayan range.

So, only two of the above are found in the Himalayas as well as the Western Ghats.
Therefore, option (b) is the correct answer.

Question 88. + 2.0 - 0.66

Consider the following statements:

Statement-I: Cirrus clouds help in the cooling of the surface of the Earth.
Statement-II: High-level thin clouds are transparent to shortwave radiation but absorb the outgoing
longwave radiation.

Which one of the following is correct in respect of the above statements?

Both Statement–I and Statement–II are correct and Statement–II is the correct
A.
explanation for Statement–I

Both Statement–I and Statement–II are correct and Statement–II is not the correct
B.
explanation for Statement–I

C. Statement–I is correct but Statement–II is incorrect

  D. Statement–I is incorrect but Statement–II is correct

Question Analytics
3765 USERS 1495 USERS 2270 USERS 114.8 SECS

ATTEMPTED  SOLVED CORRECTLY  SOLVED INCORRECTLY  YOUR TIME

40.15 SECS 0.0 SECS


 AVG. SOLVING TIME  FASTEST SOLVING TIME

Explanation :

High-level clouds occur above about 20,000 feet and are referred to as cirrus clouds. Due to
cold tropospheric temperatures at these levels, the clouds primarily are composed of ice
crystals and often appear thin, streaky, and white. They are highly transparent to shortwave
radiation but they readily absorb the outgoing longwave radiation. They then emit the
longwave radiation back to the Earth's surface. So, statement 2 is correct.
The portion of the radiation thus trapped and sent back to the Earth's surface adds to the
shortwave energy from the Sun and the longwave energy from the air already reaching the
surface. The additional energy causes a warming of the surface and atmosphere. The overall
effect of the high thin cirrus clouds then is to enhance atmospheric greenhouse warming. On
the other hand, low, thick clouds primarily reflect solar radiation and cool the surface of the
Earth. So, statement 1 is not correct.

So, Statement–I is incorrect but Statement–II is correct.

Therefore, option (d) is the correct answer.

Question 89. + 2.0 - 0.66

Which one of the following statements best describes the term 'population momentum'?

It is the phenomenon where a country witnesses an increase in young population even if


 A.
there is a decline in fertility rates.

It is a situation where the older population exceeds the population in the reproductive
 B.
age.

It is a phenomenon where the population remains stagnant due to a balance between


C.
birth and death rates.

D. It is an increasing rate of population growth as a result of declining mortality rates.

Question Analytics
3960 USERS 2382 USERS 1578 USERS 68.0 SECS

ATTEMPTED  SOLVED CORRECTLY  SOLVED INCORRECTLY  YOUR TIME

38.55 SECS 0.0 SECS

 AVG. SOLVING TIME  FASTEST SOLVING TIME


Explanation :

Population momentum is the tendency of a highly fertile population that has been
increasing rapidly in size to continue to do so for decades after the onset of even a substantial
decline in fertility. This results from the youthful age structure of such a population.
These populations contain large numbers of children who have still to grow into adulthood and
the years of reproduction. Thus, even a dramatic decline in fertility, which affects only the
numbers at age zero, cannot prevent the continuing growth of the number of adults of
childbearing age for at least two or three decades.
Eventually, as these large groups pass through the childbearing years to middle and older age,
the smaller numbers of children resulting from the fertility decline lead to a moderation in the
population growth rate.

Therefore, option (a) is the correct answer.

Relevance: The world’s population is expected to grow largely driven by a phenomenon called
population momentum.

Question 90. + 2.0 - 0.66

Which of the following evidence supports the theory that the Himalayas are still rising?

1. Frequent shallow earthquakes

2. Discovery of marine fossils


3. Youthful Himalayan rivers

4. Residual nature of mountains


Select the correct answer using the code given below:

A. 1 and 2 only

B. 3 and 4 only

  C. 1, 2 and 3 only

D. 1, 2, 3 and 4

Question Analytics
4324 USERS 1779 USERS 2545 USERS 69.2 SECS

ATTEMPTED  SOLVED CORRECTLY  SOLVED INCORRECTLY  YOUR TIME

45.9 SECS 0.0 SECS


45.9 SECS 0.0 SECS

 AVG. SOLVING TIME  FASTEST SOLVING TIME

Explanation :

Due to the fierce pressure, the Indian plate is slowly being pushed below the Eurasian Plate.
This built-up pressure is released by sudden movements which take place deep inside the
Earth's crust. It releases a huge amount of energy sending shock waves to the surface resulting
in earthquakes. The swarms of earthquakes and periods of quietude occur just before a
giant earthquake strikes the region showing that the Himalayas are still rising. So, point 1 is
correct.
The discovery of a hippopotamus skull at a high altitude in Ladakh indicates that till
recently the Himalayan region was a warm, low-lying swampy region. The findings revealed
that their presence at such heights highlights the tectonic forces that lifted these sediments
from a near-coastal marine environment to these heights. Fossils of marine animals
discovered at high altitudes in the Himalayas indicate that these locations were once aquatic
and submerged throughout the early stages of mountain range construction. Their current
height above sea level suggests a significant uplift since fossilisation, indicating continued
vertical migration. So, point 2 is correct.
Evidence of Stone Age man in the higher reaches shows that the mountains' rapid rise
was more recent. It could have been witnessed by primitive man.
The Himalayan rivers are still in their (geomorphologically) youthful stage, despite their
great age. This ever-youthfulness of the Himalayan rivers is an inherent character due to the
continuing uplift of the terrains through which the rivers flow. As the Himalayan terrain rose
progressively, the rivers kept entering their courses deeper and deeper. Over the long period of
millions of years, deep gorges or canyons with nearly vertical walls developed in the stretches
of impediments. Where the mountain barrier rose much faster and the rivers failed to keep
pace, they dropped in waterfalls and cascades. So, point 3 is correct.
Himalayas are fold mountains. Fold mountains are created where two or more of Earth's
tectonic plates are pushed together. At these colliding, compressing boundaries, rocks and
debris are warped and folded into rocky outcrops, hills, mountains, and entire mountain ranges.
Fold mountains are created through a process called orogeny. Aravalli is a type of residual
mountain. So, point 4 is not correct.

Therefore, option (c) is the correct answer.

Knowledge Box
Hippopotamus flourished in India until the end of the Pleistocene era. They may have
disappeared owing to climate change, rapidly depleting water sources and the disappearance
of vegetation. It is estimated that the last glacial period ending 14,000 years ago brought these
changes in the climate.
There is no mention of hippopotamus anywhere in ancient Indian sculpture, cave art,
architecture, folk art or folklore.

Relevance: According to researchers, the Himalaya is a fragile mountain range that is still rising as
the Indian plate continues to push into the Asian plate.

Question 91. + 2.0 - 0.66

Which one of the following, famously known as the Pink Lake, is on the verge of extinction due to
high levels of pollution?

A. Lake Titicaca

B. Lake Chad

  C. Lake Retba

D. Lake Volta

Question Analytics
3153 USERS 1435 USERS 1718 USERS 37.9 SECS

ATTEMPTED  SOLVED CORRECTLY  SOLVED INCORRECTLY  YOUR TIME

21.44 SECS 0.0 SECS

 AVG. SOLVING TIME  FASTEST SOLVING TIME

Explanation :

Lake Retba, more commonly referred to as the Pink Lake, is located around 35km from the
city of Dakar, Senegal. The lake is isolated from the sea by sand dunes. Its fresh water comes
from the seasonal water table in the dunes, which are higher than the lake. The lake’s waters
are virtually devoid of life, except for a few microscopic algae and bacteria.
The Lake had extremely high levels of nitrates —chemical compounds —in May 2023:
12,491.71 to 15,394.75 mg/l compared with 5.82 to 12.05 mg/l in December 2022. The
nitrate limit set by the World Health Organization (WHO) is 50 mg/l for water to be considered
potable. This sudden increase in nitrate levels is believed to be associated with the
establishment of hotels near the lake and ponds, and the rapid, uncontrolled urbanisation of this
northern part of the lake, which has no sewage system.
The pink colouration of the lake is due to the proliferation of halophilic green algae (living in a
salty environment), Dunaliella salina, which contains red pigments. The alga is associated
with halophilic bacteria of the genus Halobacterium. This microscopic alga’s resistance to salt
comes from its high concentration of carotenoid pigments, which protect it from light, and its
high glycerol content.

Therefore, option (c) is the correct answer.

Knowledge Box

Lake Titicaca: It is the largest freshwater lake in South America. It is located high in the
Andes mountains between Peru and Bolivia, divided by the Tiquina Strait.
Lake Chad: It is a vast area of fresh water located in the middle of dunes which covers
territories in 4 countries: Nigeria, Cameroon, Niger and Chad. This lake has been
constantly evolving with respect to environmental conditions.
Lake Volta: It is an artificial lake in Ghana. The lake is formed by the Akosombo Dam,
which, begun in 1961 and completed in 1965, dammed the Volta River just south of Ajena and
created a lake extending upstream from the Akosombo Dam to Yapei, beyond the former
confluence of the Black Volta and White Volta rivers.

Relevance: Lake Retba’s waters are virtually devoid of life and are on the verge of disappearing due
to pollution and mining.

Question 92. + 2.0 - 0.66

Arrange the following cities located around the South China Sea in North to South direction:

1. Taipei
2. Ho Chi Minh City

3. Manila
4. Singapore

Select the correct answer using the code given below:

 A. 1-2-3-4

 B. 1-3-2-4
C. 2-1-3-4

D. 4-3-1-2

Question Analytics
3573 USERS 1219 USERS 2354 USERS 76.4 SECS

ATTEMPTED  SOLVED CORRECTLY  SOLVED INCORRECTLY  YOUR TIME

42.25 SECS 0.0 SECS

 AVG. SOLVING TIME  FASTEST SOLVING TIME

Explanation :

Taipei is a special municipality and seat of the government of Taiwan (Republic of China).
Manila is the capital and chief city of the Philippines. The city is the centre of the country’s
economic, political, social, and cultural activity. It is located on the island of Luzon and
spreads along the eastern shore of Manila Bay at the mouth of the Pasig River.
Ho Chi Minh City is the largest city in Vietnam. The city lies along the Saigon River to the
north of the Mekong River delta, about 50 miles (80 km) from the South China Sea.
Singapore is a city-state located at the southern tip of the Malay Peninsula. It consists of the
diamond-shaped Singapore Island and some 60 small islets. The main island is separated from
Peninsular Malaysia to the north by Johor Strait.

So, the correct chronological order of the above cities from North to South direction is 1-3-2-4.

Therefore, option (b) is the correct answer.


Relevance: Recently, the Philippines and Vietnam agreed to expand cooperation in the South China
Sea.

Question 93. + 2.0 - 0.66

A lower amount of carbon dioxide (CO2) gas in the atmosphere of a planet compared to that of its
neighbours can indicate:

 A. absence of habitability

 B. presence of liquid water

C. terrestrial nature of the planet

D. increased volcanic activity

Question Analytics
4193 USERS 1164 USERS 3029 USERS 39.9 SECS

ATTEMPTED  SOLVED CORRECTLY  SOLVED INCORRECTLY  YOUR TIME

51.19 SECS 0.0 SECS

 AVG. SOLVING TIME  FASTEST SOLVING TIME

Explanation :

Habitability is a theoretical astronomical concept that means a celestial body is capable of


hosting and retaining liquid water on its surface. The habitable zone is sometimes referred to as
the Goldilocks zone.
Recently, an international team of researchers has found that a lower amount of Carbon
Dioxide (CO₂) gas in the atmosphere of a planet compared to that of its neighbours could hint
at the presence of liquid water on that planet. It is being claimed that it can be used as a new
'habitability signature' and that it is a “practical method for detecting habitability”. CO₂ is a
strong absorber in the infrared. By comparing the amount of CO₂ in different planets’
atmospheres, it can be used to identify those planets with oceans, which make them more
likely to be able to support life.
Life on Earth accounts for 20 percent of the total amount of captured CO₂, with the rest
mainly being absorbed by the oceans. On another planet, this number could be much larger.
Oxygen can transform into ozone and it turns out ozone has a detectable signature right next
to CO₂.
Volcanic eruptions lead to large amounts of CO₂ emissions. Venus is a terrestrial planet and
is composed of more than 96 per cent CO₂.

Therefore, option (b) is the correct answer.

Relevance: Recently, an international team of researchers has developed a new method to detect the
potential habitability of exoplanets using carbon dioxide levels.

Question 94. + 2.0 - 0.66

Consider the following:


1. Gallium arsenide

2. Silicon
3. Boron

4. Germanium

5. Neon
6. Diamond

How many of the above elements can be used as semiconductors?

A. Only three

B. Only four

  C. Only five

D. All six

Question Analytics
3827 USERS 670 USERS 3157 USERS 22.3 SECS

ATTEMPTED  SOLVED CORRECTLY  SOLVED INCORRECTLY  YOUR TIME

26.02 SECS 0.0 SECS


 AVG. SOLVING TIME  FASTEST SOLVING TIME

Explanation :

A semiconductor is a substance that has specific electrical properties that enable it to serve
as a foundation for computers and other electronic devices. It is typically a solid chemical
element or compound that conducts electricity under certain conditions but not others. This
makes it an ideal medium to control electrical current and everyday electrical appliances. They
have properties that sit between the conductor and insulator.
Gallium arsenide, germanium and silicon are some of the most commonly used
semiconductors. Silicon is used in the fabrication of electronic circuits. So, points 1, 2 and 4
are correct.
Gallium arsenide is a semiconductor compound used in some diodes, field-effect
transistors (FETs), and integrated circuits (ICs).
Elemental semiconductors include antimony, arsenic, boron, carbon, selenium, sulphur
and tellurium. Among wide bandgap semiconductors, diamond is considered to be the
ultimate semiconductor for applications in high-power electronics due to its exceptional
properties. So, points 3 and 6 are correct.
Neon is important to manufacture semiconductor chips but is not present in the chips. It
doesn't directly touch the silicon during manufacturing. It helps make the Deep Ultraviolet
(DUV) light used in the photolithographic process that patterns semiconductors. So, point 5 is
not correct.

So, only five of the above elements are used as semiconductors.

Therefore, option (c) is the correct answer.


Relevance: Recently, a novel laser slicing technique was invented for diamond semiconductors.

Question 95. + 2.0 - 0.66

In biotechnology, ‘DnaA protein’ is used for:

 A. initiation of the DNA replication

B. joining breaks in the DNA

C. identification of DNA polymorphism

D. arranging the DNA into specific sequences


Question Analytics
2918 USERS 1123 USERS 1795 USERS 7.3 SECS

ATTEMPTED  SOLVED CORRECTLY  SOLVED INCORRECTLY  YOUR TIME

26.29 SECS 0.0 SECS

 AVG. SOLVING TIME  FASTEST SOLVING TIME

Explanation :

The dnaA gene which encodes for the DnaA protein is considered a cold-inducible protein and
possesses both DNA binding/replication initiator properties and acts as a global regulator of
transcription. The DnaA protein is centrally involved in the initiation of chromosomal and
mini-chromosomal DNA replication and appears to be important in the timing control of cell-
cycle initiation. It also autoregulates the dnaA gene and influences cell membrane structural
properties.

Therefore, option (a) is the correct answer.

Relevance: Recently, scientists have uncovered how DnaA, the master key to DNA replication,
opens the door to bacterial growth.

Question 96. + 2.0 - 0.66

The term ‘Ergosphere Zone’, often seen in the news, refers to:

A. a region where the impact of solar flares is the strongest

 B. boundary that marks the end of habitable zone in space

 C. a region in black hole where an object can enter and exit

D. point in space where matter moves in a perfect spiral due to earth's gravity

Question Analytics
3217 USERS 1629 USERS 1588 USERS 31.1 SECS

ATTEMPTED  SOLVED CORRECTLY  SOLVED INCORRECTLY  YOUR TIME

29.15 SECS 0.0 SECS

 AVG. SOLVING TIME  FASTEST SOLVING TIME

Explanation :
The formation of a black hole occurs when a massive star exhausts its nuclear fuel, leading to a
supernova explosion. The remaining core implodes under its gravitational weight, creating a

black hole.
Rotating black holes, also known as Kerr black holes, possess a distinctive feature known as
the ergosphere, which lies outside their outer event horizon.
Singularity is the point of infinite density and infinitesimal volume, at which space and time
become infinitely distorted according to the theory of General Relativity.
The event horizon delineates a sphere around the singularity. Once an object crosses this
sphere, escape becomes impossible unless it somehow surpasses the speed of light, an
impossibility in classical physics. Just beyond this boundary, a rotating black hole exhibits an
ergosphere—an outer sphere where an object can enter and exit if it attains sufficient
speed, but still less than the speed of light.

Therefore, option (c) is the correct answer.

Relevance: Recently scientists have suggested sending an object into the ergosphere of a black hole.

Question 97. + 2.0 - 0.66

Consider the following statements with reference to solid-state batteries:

1. They are highly stable as they are resistant to changes in temperature.


2. Unlike lithium-ion batteries, they can be charged faster.

3. They have a lower risk of fire as they do not contain flammable organic solvents.
4. They can potentially be used in space devices.

How many of the above statements are correct?

A. Only one

B. Only two

 C. Only three

 D. All four

Question Analytics
3031 USERS 1405 USERS 1626 USERS 50.9 SECS

ATTEMPTED  SOLVED CORRECTLY  SOLVED INCORRECTLY  YOUR TIME

34.03 SECS 0.0 SECS

 AVG. SOLVING TIME  FASTEST SOLVING TIME

Explanation :

A solid-state battery is a battery technology that uses a solid electrolyte instead of a liquid
electrolyte. They are characterised by being highly stable because they are resistant to
changes in temperature and can robustly endure high temperatures and high voltages. So,
statement 1 is correct.
Lithium-ion batteries typically have slower charging rates due to limitations in ion mobility
within the liquid electrolyte. Solid-state batteries have the potential for faster charging due
to improved ion conductivity in the solid electrolytes, allowing for quicker charge transfer. So,
statement 2 is correct.
One of the expected applications for solid-state batteries is electric vehicles. Currently, electric
vehicles use lithium-ion batteries. However, if they use solid-state batteries, the risk of ignition
due to accidents is expected to decrease since they do not contain flammable organic
solvents. So, statement 3 is correct.
It is also anticipated that use of solid-state batteries will include electronic device backup
power supplies and Internet of Things (IoT) sensors. If used on PCs or smartphones, they
should enable powerful operation for a longer time. Furthermore, they can be expected to be
used in aeroplanes and ships. Moreover, as they are resistant to temperature changes across
the spectrum from high to low temperature, it can be expected that their applications will
expand to include devices used in outer space. So, statement 4 is correct.

So, all four of the above statements are correct.


Therefore, option (d) is the correct answer.

Knowledge Box

Relevance: Recently, Toyota announced plans to roll out a solid-state battery-powered electric
vehicle.

Question 98. + 2.0 - 0.66

“Mars solar conjunction” is a phenomenon that occurs when:

 A. Sun is between Earth and Mars

B. Sun is positioned nearest to Mars

C. Earth is between Mars and Sun

 D. Sun, Mars and its moons align in a straight line


Question Analytics
3539 USERS 1013 USERS 2526 USERS 61.7 SECS

ATTEMPTED  SOLVED CORRECTLY  SOLVED INCORRECTLY  YOUR TIME

32.8 SECS 0.0 SECS

 AVG. SOLVING TIME  FASTEST SOLVING TIME

Explanation :

Mars solar conjunction is a phenomenon which occurs when the Sun is between Earth and
Mars. It happens once every two years. The Sun expels hot, ionized gas from its corona, which
extends far into space.
During solar conjunction, this gas can interfere with radio signals when engineers try to
communicate with spacecraft at Mars (Red Planet), corrupting commands and resulting in
unexpected behaviour from deep space explorers.
Mars opposition occurs when Earth "catches up" to Mars and is positioned between the Sun
and the Red Planet. During Mars opposition from Earth, one can see Mars rise in the east as
the sun sets in the west, with the Red Planet being visible in the skies all night long. Such
opposition occurs roughly every 26 months.

Therefore, option (a) is the correct answer.

Relevance: Recently, NASA announced that it will stop sending commands to its fleet of robotic
probes on or orbiting the red planet Mars due to Mars's solar conjunction.

Question 99. + 2.0 - 0.66

Consider the following statements:

Statement-I: Multiple Independently Targetable Re-entry Vehicle (MIRV) reduces the risk of
interception by an anti-ballistic missile system.

Statement-II: MIRV allows a missile to carry multiple nuclear warheads which can be independently
targeted to different locations.

Which one of the following is correct in respect of the above statements?

Both Statement–I and Statement–II are correct and Statement–II is the correct
 A.
explanation for Statement–I

Both Statement–I and Statement–II are correct and Statement–II is not the correct
B.
explanation for Statement–I
C. Statement–I is correct but Statement–II is incorrect

D. Statement–I is incorrect but Statement–II is correct

Question Analytics
3644 USERS 2791 USERS 853 USERS 11.1 SECS

ATTEMPTED  SOLVED CORRECTLY  SOLVED INCORRECTLY  YOUR TIME

27.66 SECS 0.0 SECS

 AVG. SOLVING TIME  FASTEST SOLVING TIME

Explanation :

Multiple Independently Targetable Re-entry Vehicle (MIRV) technology consists of


several reentry vehicles, each equipped to carry 2–10 nuclear warheads. These warheads can
be designated for various targets, spaced hundreds of kilometers apart. Alternatively,
multiple warheads can target a single location. So, statement 2 is correct.
Deployment of MIRV can enhance first-strike proficiency for strategic forces and also provide
greater target damage for a given thermonuclear weapon payload. Moreover, as it can release
multiple warheads, it serves the purpose of a cluster munition, as it reduces the number of
missiles and launch facilities required. With the MIRV, the effectiveness of an anti-ballistic
missile system that relies on intercepting individual warheads is also reduced. So,
statement 1 is correct.

So, both Statement–I and Statement–II are correct and Statement–II is the correct explanation
for Statement–I.

Therefore, option (a) is the correct answer.


Relevance: Recently, the first flight test of Made-in-India Agni-5 missile with Multiple
Independently Targetable Re-entry Vehicle (MIRV) technology was conducted.

Question 100. + 2.0 - 0.66

‘Phthalates, Octoxynols and Nonoxynols’, causes of concern in recent times, are most likely to be
found in which one of the following?

A. Antibiotics

 B. Performance-enhancing drugs
 C. Perfumes

D. Food preservatives

Question Analytics
3372 USERS 931 USERS 2441 USERS 65.1 SECS

ATTEMPTED  SOLVED CORRECTLY  SOLVED INCORRECTLY  YOUR TIME

40.04 SECS 0.0 SECS

 AVG. SOLVING TIME  FASTEST SOLVING TIME

Explanation :

Phthalates, Octoxynols and Nonoxynols are chemicals widely used in the manufacture of
counterfeit perfumes and fragrances.

Phthalates are typically employed as solvents and stabilisers in perfumes. They are known
hormone disruptors linked to birth defects in male infants. Octoxynols and nonoxynols are
also concerning as they can break down into persistent hormone disruptors.
The usage of such inferior and possibly adulterated cosmetics can lead to a spectrum of
adverse health reactions, ranging from mild skin irritations to severe conditions such as
anaphylactic shock and even organ failure. Due to their low biodegradability, they persist in
the environment, especially in aquatic ecosystems and can encourage bioaccumulation.

Therefore, option (c) is the correct answer.

Relevance: BAN Toxics, a chemical and waste management advocacy group, has issued a warning
against the alarming rise in the sale of counterfeit perfumes.

You might also like